Сборник для подготовки к ЕГЭ (базовый уровень).Прототип задания № 19

Сборник для подготовки к ЕГЭ (базовый уровень)

Прототип задания № 19

1. Сумма цифр трёхзначного на­ту­раль­но­го числа А де­лит­ся на 12. Сумма цифр числа (А + 6) также де­лит­ся на 12. Най­ди­те наи­мень­шее воз­мож­ное число А.

2. Найдите трёхзначное на­ту­раль­ное число, боль­шее 400, ко­то­рое при де­ле­нии на 6 и на 5 даёт рав­ные не­ну­ле­вые остат­ки и пер­вая слева цифра ко­то­ро­го яв­ля­ет­ся сред­ним ариф­ме­ти­че­ским двух дру­гих цифр. В от­ве­те ука­жи­те какое-нибудь одно такое число.

3. Сумма цифр трёхзначного числа A де­лит­ся на 13. Сумма цифр числа A+5 также де­лит­ся на 13. Най­ди­те такое число A.

4. Найдите трёхзначное число A, об­ла­да­ю­щее всеми сле­ду­ю­щи­ми свойствами:

· сумма цифр числа A де­лит­ся на 4;

· сумма цифр числа (A + 2) де­лит­ся на 4;

· число A боль­ше 200 и мень­ше 400.

В от­ве­те ука­жи­те какое-нибудь одно такое число.

5. Приведите при­мер трёхзначного на­ту­раль­но­го числа, крат­но­го 4, сумма цифр ко­то­ро­го равна их произведению. В от­ве­те ука­жи­те ровно одно такое число.

6. Вычеркните в числе 23462141 три цифры так, чтобы по­лу­чив­ше­е­ся число де­ли­лось на 12. В от­ве­те ука­жи­те ровно одно по­лу­чив­ше­е­ся число.

7. Найдите пя­ти­знач­ное число, крат­ное 25, со­сед­ние цифры ко­то­ро­го от­ли­ча­ют­ся на 2. В от­ве­те ука­жи­те какое-нибудь одно такое число.

8. Найдите четырёхзначное число, крат­ное 22, про­из­ве­де­ние цифр ко­то­ро­го равно 24. В от­ве­те ука­жи­те какое-нибудь одно такое число.

9. Найдите пятизначное число, кратное 12, любые две соседние цифры которого отличаются на 2. В ответе укажите какое-нибудь одно такое число.

10. Найдите четырёхзначное число, крат­ное 22, про­из­ве­де­ние цифр ко­то­ро­го равно 40. В от­ве­те ука­жи­те какое-нибудь одно такое число.

11. Цифры четырёхзначного числа, крат­но­го 5, за­пи­са­ли в об­рат­ном по­ряд­ке и по­лу­чи­ли вто­рое четырёхзначное число. Затем из пер­во­го числа вычли вто­рое и по­лу­чи­ли 3627. При­ве­ди­те ровно один при­мер та­ко­го числа.

12. Найти че­ты­рех­знач­ное число, крат­ное 44, любые две со­сед­ние цифры ко­то­ро­го от­ли­ча­ют­ся на 1. В от­ве­те ука­жи­те любое такое число.

13. Найдите чётное трёхзначное натуральное число, сумма цифр которого на 1 меньше их произведения. В ответе укажите какое-нибудь одно такое число.

14. Трёхзначное число при де­ле­нии на 10 даёт в остат­ке 3. Если по­след­нюю цифру числа пе­ре­не­сти в на­ча­ло его записи, то по­лу­чен­ное число будет на 72 боль­ше первоначального. Най­ди­те ис­ход­ное число.

15. Сумма цифр трёхзначного на­ту­раль­но­го числа А де­лит­ся на 12. Сумма цифр числа (А + 6) также де­лит­ся на 12. Най­ди­те наи­мень­шее число А, удо­вле­тво­ря­ю­щее усло­вию А  700.

16. Вычеркните в числе 23462141 три цифры так, чтобы по­лу­чив­ше­е­ся число де­ли­лось на 12. В от­ве­те ука­жи­те какое-нибудь одно такое число.

17. Цифры четырёхзначного числа, крат­но­го 5, за­пи­са­ли в об­рат­ном по­ряд­ке и по­лу­чи­ли вто­рое четырёхзначное число. Затем из пер­во­го числа вычли вто­рое и по­лу­чи­ли 1458. При­ве­ди­те ровно один при­мер та­ко­го числа.

18. Найдите ше­сти­знач­ное на­ту­раль­ное число, ко­то­рое за­пи­сы­ва­ет­ся толь­ко циф­ра­ми 1 и 0 и де­лит­ся на 24.

19. Найдите пя­ти­знач­ное число, крат­ное 15, про­из­ве­де­ние цифр ко­то­ро­го равно 60. В от­ве­те ука­жи­те какое-нибудь одно такое число.

Укажите наи­мень­шее такое число.

20. Найдите трёхзначное на­ту­раль­ное число, боль­шее 500, ко­то­рое при де­ле­нии на 8 и на 5 даёт рав­ные не­ну­ле­вые остат­ки и сред­няя цифра ко­то­ро­го яв­ля­ет­ся сред­ним ариф­ме­ти­че­ским край­них цифр. В от­ве­те ука­жи­те какое-нибудь одно такое число.

21. Найдите четырёхзначное число, крат­ное 45, все цифры ко­то­ро­го различны и чётны. В от­ве­те укажите какое-нибудь одно такое число.

22. Найдите четырёхзначное число, крат­ное 22, про­из­ве­де­ние цифр ко­то­ро­го равно 60. В от­ве­те ука­жи­те какое-нибудь одно такое число.

23. Приведите при­мер четырёхзначного числа, крат­но­го 12, про­из­ве­де­ние цифр ко­то­ро­го равно 10. В от­ве­те ука­жи­те ровно одно такое число.

24. Найдите наи­мень­шее пя­ти­знач­ное число, крат­ное 55, про­из­ве­де­ние цифр ко­то­ро­го боль­ше 50, но мень­ше 75.

25. Найдите трёхзначное на­ту­раль­ное число, ко­то­рое при де­ле­нии на 4 и на 15 даёт рав­ные не­ну­ле­вые остат­ки и сред­няя цифра ко­то­ро­го яв­ля­ет­ся сред­ним ариф­ме­ти­че­ским край­них цифр. В от­ве­те ука­жи­те какое-нибудь одно такое число.

26. Найдите трёхзначное число A, об­ла­да­ю­щее всеми сле­ду­ю­щи­ми свойствами:

· сумма цифр числа A де­лит­ся на 5;

· сумма цифр числа (A + 4) де­лит­ся на 5;

· число A боль­ше 350 и мень­ше 400.

В от­ве­те ука­жи­те какое-нибудь одно такое число.

27. Вычеркните в числе 181615121 три цифры так, чтобы по­лу­чив­ше­е­ся число де­ли­лось на 12. В от­ве­те ука­жи­те какое-нибудь одно такое число.

28. Найдите пятизначное натуральное число, кратное 3, сумма цифр которого равна их произведению. В ответе укажите какое-нибудь одно такое число.

29. Найдите четырёхзначное на­ту­раль­ное число, крат­ное 19, сумма цифр ко­то­ро­го на 1 боль­ше их произведения.

30. Найдите пятизначное натуральное число, кратное 5, сумма цифр которого равна их произведению. В ответе укажите какое-нибудь одно такое число.

31. Вычеркните в числе 53164018 три цифры так, чтобы по­лу­чив­ше­е­ся число де­ли­лось на 15. В от­ве­те ука­жи­те ровно одно по­лу­чив­ше­е­ся число.

32. Приведите при­мер трёхзначного на­ту­раль­но­го числа, боль­ше­го 500, ко­то­рое при де­ле­нии на 8 и на 5 даёт рав­ные не­ну­ле­вые остат­ки и пер­вая слева цифра ко­то­ро­го яв­ля­ет­ся сред­ним ариф­ме­ти­че­ским двух дру­гих цифр. В от­ве­те ука­жи­те ровно одно такое число.

33. Приведите при­мер четырёхзначного числа, крат­но­го 15, про­из­ве­де­ние цифр ко­то­ро­го боль­ше 35, но мень­ше 45. В от­ве­те ука­жи­те ровно одно такое число.

34. Найдите ше­сти­знач­ное на­ту­раль­ное число, ко­то­рое за­пи­сы­ва­ет­ся толь­ко циф­ра­ми 1 и 5 и де­лит­ся на 45. В от­ве­те ука­жи­те какое-нибудь одно такое число.

Укажите наибольшее такое число.

35. Найдите трёхзначное натуральное число, большее 500, которое при делении на 8 и на 5 даёт равные ненулевые остатки и средняя цифра которого является средним арифметическим крайних цифр. В ответе укажите какое-нибудь одно такое число.

36. Найдите трёхзначное натуральное число, большее 600, которое при делении и на 3, и на 4, и на 5 даёт в остатке 1 и цифры в записи которого расположены в порядке убывания слева направо. В ответе укажите какое-нибудь одно такое число.

37. Найдите трёхзначное натуральное число, большее 500, которое при делении и на 6, и на 5 даёт равные ненулевые остатки и средняя цифра в записи которого является средним арифметическим крайних цифр. В ответе укажите какое-нибудь одно такое число.

38. Вычеркните в числе 85417627 три цифры так, чтобы по­лу­чив­ше­е­ся число де­ли­лось на 18. В от­ве­те ука­жи­те ровно одно по­лу­чив­ше­е­ся число.

39. Найдите четырёхзначное число, крат­ное 88, все цифры ко­то­ро­го раз­лич­ны и чётны. В от­ве­те ука­жи­те какое-нибудь одно такое число.

40. Найдите трех­знач­ное на­ту­раль­ное число, боль­шее 500, ко­то­рое при де­ле­нии на 4, на 5 и на 6 дает в остат­ке 2, и в за­пи­си ко­то­ро­го есть толь­ко две раз­лич­ные цифры. В от­ве­те ука­жи­те какое-нибудь одно такое число.

41. Найдите на­ту­раль­ное число, боль­шее 1340, но мень­шее 1640, ко­то­рое де­лит­ся на каж­дую свою цифру и все цифры ко­то­ро­го раз­лич­ны и не равны нулю. В от­ве­те ука­жи­те какое-нибудь одно такое число.

42. Найдите трёхзначное натуральное число, которое при делении и на 4, и на 5, и на 6 даёт в остатке 2 и цифры в записи которого чётные. В ответе укажите какое-нибудь одно такое число.

43. Приведите при­мер трёхзначного на­ту­раль­но­го числа, боль­ше­го 500, ко­то­рое при де­ле­нии на 3, на 4 и на 5 даёт в остат­ке 2 и в за­пи­си ко­то­ро­го есть толь­ко две раз­лич­ные цифры. В от­ве­те ука­жи­те ровно одно такое число.

44. Цифры четырёхзначного числа, крат­но­го 5, за­пи­са­ли в об­рат­ном по­ряд­ке и по­лу­чи­ли вто­рое четырёхзначное число. Затем из пер­во­го числа вычли вто­рое и по­лу­чи­ли 4536. При­ве­ди­те ровно один при­мер та­ко­го числа.

45. Найдите че­ты­рех­знач­ное число, крат­ное 66, все цифры ко­то­ро­го раз­лич­ны и четны. В от­ве­те ука­жи­те какое-нибудь такое число.

46. Приведите при­мер трёхзначного на­ту­раль­но­го числа, ко­то­рое при де­ле­нии на 4 и на 15 даёт рав­ные не­ну­ле­вые остат­ки и пер­вая спра­ва цифра ко­то­ро­го яв­ля­ет­ся сред­ним ариф­ме­ти­че­ским двух дру­гих цифр. В от­ве­те ука­жи­те ровно одно такое число.

47. Приведите при­мер трёхзначного числа А, об­ла­да­ю­ще­го сле­ду­ю­щи­ми свойствами:

1) сумма цифр числа А де­лит­ся на 6;

2) сумма цифр числа (А + 3) также де­лит­ся на 6;

3) число А боль­ше 350 и мень­ше 400.

В от­ве­те ука­жи­те ровно одно такое число.

48. Приведите при­мер четырёхзначного на­ту­раль­но­го числа, крат­но­го 4, сумма цифр ко­то­ро­го равна их произведению. В от­ве­те ука­жи­те ровно одно такое число.

49. Найдите трёхзначное число, кратное 11, все цифры которого различны, а сумма квадратов цифр делится на 4, но не делится на 16. В ответе укажите какое-нибудь одно такое число.

50. Найдите трёхзначное натуральное число, которое при делении и на 3, и на 5, и на 7 даёт в остатке 2 и в записи которого есть только две различные цифры. В ответе укажите какое-нибудь одно такое число.

Ответы к прототипу задания № 19

  1. Ответ: 699

  2. Ответ: 453|573|693

  3. Ответ: 899

  4. Ответ: 299|398

  5. Ответ: 132|312

  6. Ответ: 23424|24624

  7. Ответ:13575|53575|57575|97575|57975|97975

  8. Ответ: 2134|4312|1342|3124

  9. Ответ:42024|46464|42420|42468|86424|86868

  10. Ответ:1452|1254|5412|5214|1518

  11. Ответ: 8925|8815|8705

  12. Ответ:1012|3432|5456|3212|1232|5676|7876|7656

  13. Ответ: 124|142|214|412

  14. Ответ: 253

  15. Ответ: 798

  16. Ответ: 24624|23424

  17. Ответ: 7065|7175|7285|7395

  18. Ответ: 111000

  19. Ответ: 11265

  20. Ответ: 642|963

  21. Ответ:4680|4860|6480|6840|846|8640

  22. Ответ: 5126|2156|6512|1562

  23. Ответ: 1152|1512|5112

  24. Ответ: 11275

  25. Ответ: 123|543|963

  26. Ответ: 357|366|389

  27. Ответ:181512|116112|811512|181152

  28. Ответ:11133|11313|13113|31113|33111|31311|31131|13311|13131|11331

  29. Ответ: 3211

  30. Ответ:11125|12115|11215|21115

  31. Ответ: 53160|53640

  32. Ответ: 564|684

  33. Ответ:1185|1815|8115|1245|1425|2145|2415|4125|4215

  34. Ответ: 551115

  35. Ответ: 642|963

  36. Ответ: 721|841|961

  37. Ответ: 543|753|963

  38. Ответ: 84762|85176|54162

  39. Ответ: 2640|6248|8624

  40. Ответ: 662|722

  41. Ответ: 1362|1395|1368|1632

  42. Ответ:242|422|482|602|662|842

  43. Ответ: 662|722

  44. Ответ: 9605|9715|9825|9935

  45. Ответ:2640|2046|6204|6402|4026|2046|4620

  46. Ответ: 243|423|603

  47. Ответ: 369|378|387

  48. Ответ: 1412|4112|1124

  49. Ответ: 264|286|462|682

  50. Ответ: 212|422|737

Внимательно прочитайте текст задания и выберите верный ответ из списка

Задание 3755

Проанализируйте график зависимости длины тела личинки насекомого от времени.

 

 

Выберите утверждения, которые можно сформулировать на основании анализа представленных данных.

Ответы:

Рост личинки происходит скачкообразно. — Правильный ответ

У личинки в процессе роста происходят внутренние изменения в строении.

Длина тела личинки на большинстве стадий развития увеличивается в интервале 0,4 – 0,6 см. — Правильный ответ

Насекомые живут 46 дней.

Рост личинки сопровождается линькой.

Базовый уровень ЕГЭ 2015 с WolframAlpha

2015 БАЗОВЫЙ УРОВЕНЬ 19 

ТРЕБОВАНИЯ
Уметь выполнять вычисления и преобразования
1.1  Выполнять арифметические действия, сочетая устные и письменные приемы; находить значения корня натуральной степени, степени с рациональным показателем, логарифма
ЭЛЕМЕНТЫ СОДЕРЖАНИЯ 

1.4.1  Преобразования выражений, включающих арифметические операции
1.4.2  Преобразования выражений, включающих операцию возведения в степень

01 ТРЕХЗНАЧНЫЕ ЧИСЛА: ДВА ДЕЛИТЕЛЯ

Приведите пример трёхзначного натурального числа большего 500, которое при делении на 6 и на 5 даёт равные ненулевые остатки и средняя цифра которого является средним арифметическим крайних цифр. В ответе укажите ровно одно такое число.


02 ТРЕХЗНАЧНЫЕ ЧИСЛА: ТРИ ДЕЛИТЕЛЯ

Найдите трёхзначное натуральное число, которое при делении на 4, на 5 и на 6 даёт в остатке 2 и цифры которого четные. В ответе укажите какое-нибудь одно такое число.

03 ТРЕХЗНАЧНЫЕ ЧИСЛА: ТРИ ДЕЛИТЕЛЯ
Найдите трехзначное натуральное число, большее 400, но меньшее 650, которое делится на каждую свою цифру и все цифры которого различны и не равны нулю. В ответе укажите какое-нибудь одно такое число.

04 ТРЕХЗНАЧНЫЕ ЧИСЛА: ТРИ ДЕЛИТЕЛЯ
Найдите треханачное натуральное число, большее 500, которое при делении на 8 и на 5 даёт равные ненулевые остатки и средняя цифра которого является средним арифметическим крайних цифр. В ответе укажите какое-нибудь одно такое число.

05 ТРЕХЗНАЧНЫЕ ЧИСЛА: КРАТНОСТЬ 4
Приведите пример трёхзначного натурального числа, кратного 4, сумма цифр которого равна их произведению. В ответе укажите ровно одно такое число.

06 ТРЕХЗНАЧНЫЕ ЧИСЛА: КРАТНОСТЬ 4 И 15
Приведите пример трёхзначного натурального числа, которое при делении на 4 и на 15 даёт равные ненулевые остатки и первая справа цифра которого является средним арифметическим двух других цифр. В ответе укажите ровно одно такое число.

07 ТРЕХЗНАЧНЫЕ ЧИСЛА: СУММА ЦИФР
Сумма цифр трёхзначного натурального числа А делится на 4. Сумма цифр числа А + 5 также делится на 4. Найдите наименьшее такое число А, удовлетворяющее условию А > 300.

08 ТРЕХЗНАЧНЫЕ ЧИСЛА: ТРИ ДЕЛИТЕЛЯ НАИМЕНЬШЕГО ТРЕХЗНАЧНОГО ЧИСЛА ИЗ РАЗЛИЧНЫХ ЦИФР
Найдите наименьшее трёхзначное натуральное число, которое при делении на 2 даёт остаток 1, при делении на 3 даёт остаток 2, при делении на 5 даёт остаток 3 и которое записано тремя различными цифрами.

09 ТРЕХЗНАЧНЫЕ ЧИСЛА: ТРИ ДЕЛИТЕЛЯ НАИБОЛЬШЕГО ТРЕХЗНАЧНОГО ЧИСЛА ИЗ РАЗЛИЧНЫХ ЦИФР
Найдите наибольшее трёхзначное натуральное число, которое при делении на 5 и на 11 даёт равные ненулевые остатки.

10 ТРЕХЗНАЧНЫЕ ЧИСЛА: КРАТНОСТЬ 70
Найдите трёхзначное число, кратное 70, все цифры которого различны, а сумма квадратов цифр делится на 5, но не делится на 25. В ответе укажите какое-нибудь одно такое число.

11 ТРИ СВОЙСТВА ТРЕХЗНАЧНОГО ЧИСЛА
Приведите пример трёхзначного числа А, обладающего следующими свойствами:
1 :: сумма цифр числа А делится на 6;
2 :: сумма цифр числа А+3 также делится на 6;
3 :: число А больше 350 и меньше 400.
В ответе укажите ровно одно такое число.

12 ЧЕТЫРЕХЗНАЧНОЕ ЧИСЛО, КРАТНОЕ 12, С ИЗВЕСТНЫМ ПРОИЗВЕДЕНИЕМ ЦИФР
Приведите пример четырёхзначного числа, кратного 12, произведение цифр которого равно 10. В ответе укажите какое-нибудь одно такое число.

13 ЧЕТЫРЕХЗНАЧНОЕ ЧИСЛО, КРАТНОЕ 15, С ИЗВЕСТНЫМ ПРОИЗВЕДЕНИЕМ ЦИФР
Приведите пример четырёхзначного числа, кратного 15, произведение цифр которого больше 35, но меньше 45. В ответе укажите какое-нибудь одно такое число.

14 ЧЕТЫРЕХЗНАЧНОЕ ЧИСЛО, КРАТНОЕ 44
Найдите четырёхзначное число, кратное 44, любые две соседние цифры которого отличаются на 1. В ответе укажите какое-нибудь одно такое число.

15 ЧЕТЫРЕХЗНАЧНОЕ ЧИСЛО, КРАТНОЕ 66
Найдите четырехзначное число, кратное 66, все цифры которого различны и чётны. В ответе укажите какое-нибудь одно такое число.

16 ЧЕТЫРЕХЗНАЧНОЕ ЧИСЛО, КРАТНОЕ 66
Приведите пример четырёхзначного натурального числа, кратного 18, сумма цифр которого на 1 меньше их произведения. В ответе укажите ровно одно такое число.

17 ПРОИЗВЕДЕНИЕ НАИБОЛЬШЕГО ЧЕТЫРЕХЗНАЧНОГО ЧИСЛА, КРАТНОГО N
Найдите наибольшее четырёхзначное число, кратное 11, у которого произведение его цифр равно 12.

18 ПРОИЗВЕДЕНИЕ НАИМЕНЬШЕГО ЧЕТЫРЕХЗНАЧНОГО ЧИСЛА, КРАТНОГО N
Найдите наименьшее четырёхзначное число, кратное 11, у которого произведение его цифр равно 12.

19 СУММА И ПРОИЗВЕДЕНИЕ ЧЕТЫРЕХЗНАЧНОГО ЧИСЛА, КРАТНОГО N
Найдите четырёхзначное натуральное число, кратное 19, сумма цифр которого на 1 больше их произведения.

20 РАЗНОСТЬ ЧЕТЫРЕХЗНАЧНОГО ЧИСЛА, КРАТНОГО N, И ЕМУ ЗЕРКАЛЬНОГО
Цифры четырёхзначного числа, кратного 5, записали в обратном порядке и получили второе четырёхзначное число. Затем из первого числа вычли второе и получили 1458. В ответе укажите какое-нибудь одно такое число.

21 ДЕЛИМОСТЬ ПЯТИЗНАЧНОГО ЧИСЛА ИЗ УКАЗАННЫХ ЦИФР
Приведите пример пятизначного натурального числа, которое записывается только цифрами 0 и 1, делится на 20, но не делится на 40. В ответе укажите ровно одно такое число.

22 ПРОИЗВЕДЕНИЕ НАИМЕНЬШЕГО ПЯТИЗНАЧНОГО ЧИСЛА, КРАТНОГО N
Найдите наименьшее пятизначное число, кратное 11, у которого произведение его цифр равно 20.

23 ПРОИЗВЕДЕНИЕ НАИБОЛЬШЕГО ПЯТИЗНАЧНОГО ЧИСЛА, КРАТНОГО N
Найдите наибольшее пятизначное число, кратное 11, у которого произведение его цифр равно 20.

24 ДЕЛИМОСТЬ ШЕСТИЗНАЧНОГО ЧИСЛА ИЗ УКАЗАННЫХ ЦИФР
Найдите шестизначное натуральное число, которое записывается только цифрами 1 и 0 и делится на 24.

25 ПРОИЗВЕДЕНИЕ ШЕСТИЗНАЧНОГО ЧИСЛА, КРАТНОГО N
Найдите наименьшее шестизначное число, кратное 75, произведение цифр которого больше 30, но меньше 60.

26 ДОСРОЧНЫЙ ВЫЧЕРКИВАНИЕ ТРЕХ ЦИФР ШЕСТИЗНАЧНОГО ЧИСЛА, КРАТНОГО N
Вычеркните в числе 181615121 три цифры так, чтобы получившееся число делилось на 12. В ответе укажите какое-нибудь одно такое число.

 

 

Решу егэ задания 19 базовый уровень

В данной статье речь пойдёт о решении задачи 19 из варианта досрочного профильного ЕГЭ по математике, предлагавшегося для решения школьникам в 2016 году. Решение задачи 19 из ЕГЭ по математике (профильный уровень) традиционно вызывает наибольшие затруднения у выпускников, ведь это последняя, а потому обычно самая сложная задача из экзамена. По крайней мере, такое впечатление часто складывается в умах школьников, готовящихся к ЕГЭ. Но на самом деле ничего очень сложного в этих задачах нет. Посмотрите, например, как легко решается следующая задача 19 из профильного ЕГЭ по математике.

Не смущайтесь термина «хорошее» множество. Это типично для составителей вариантов ЕГЭ по математике. Когда не хватает слов, приходится использовать слова не по их прямому назначению.

Решение задачи 19 из профильного ЕГЭ по математике под буквой А

Перейдём к решению. Отвечаем на вопрос под буквой А. Является записанное множество хорошим? Предположим, что да. Если это действительно так, то это самый простой случай для нас. Ведь в этом случае требуется лишь привести пример разбиения этого множества на два множества, суммы элементов которых одинаковы. В противном случае пришлось бы доказывать принципиальную невозможность нужного разбиения. А это уже гораздо сложнее. Ну а поскольку это лишь задание под буквой А, можно надеяться, что оно достаточно простое. Итак, попытаемся разбить наше множество на два подмножества, суммы элементов в которых будут одинаковы.

К счастью, чтобы это сделать, не нужно быть Эйнштейном. Берём самое очевидное и интуитивное решение. Группируем элементы исходного множества в пары: первый с последним, второй с предпоследним и так далее:

Последняя парочка будет состоять из двух чисел: 249 и 250. Всего таких парочек получится 50. Сумма чисел в каждой парочке равна 499. А дальше берите какие угодно 25 парочек в первое множество, остальные 25 — во второе множество, и получите требуемое разбиение. Итак, ответ на вопрос под буквой А — да!

Ответ на вопрос под буквой Б из задачи 19 ЕГЭ по математике (профильный уровень)

Переходим к вопросу под буквой Б. Задание то же самое, только множество другое. Поэтому думается, что авторы-составители должны были здесь проявить оригинальность. Так что, скорее всего, это множество уже не будет хорошим. Если это так, то просто примером в данном случае ограничиться не получится, придётся всё доказывать. Ну что ж, попробуем.

Вообще говоря, если вдуматься в задание, то решение приходит само собой. Нам требуется разбить данное множество на два подмножества, суммы элементов в каждом из которых равны. Ну и, в общем, тут не нужно быть Стивином Хокингом, чтобы понять, что ключ к решению в том, чтобы найти, чему должны быть равны эти суммы! А для этого нужно посчитать сумму элементов нашего исходного множества.

Посмотрите внимательно. Перед нами классическая геометрическая прогрессия со знаменателем , первым членом и элементами. Сумма всех элементов такой прогрессии определяется по известной формуле:

Это означает, что если бы мы разбили наше множество на два подмножества с одинаковой суммой элементов в каждом из них, то эта сумма оказалась бы равной . А это нечётное число! Но ведь все элементы нашего множества — это степени двойки, то есть числа безусловно чётные. Вопрос. Может ли получиться нечётное число, если складывать чётные числа? Конечно, нет. То есть мы доказали невозможность такого разбиения. Итак, ответ к вопросу под буквой Б из решения задачи 19 из ЕГЭ по математике (профильный уровень) — нет!

Решение задачи 19 из ЕГЭ по математике (профильный уровень) под буквой В

Ну и наконец, переходим к вопросу под буквой В. Сколько же четырёхэлементных хороших множества содержится в множестве {1; 2; 4; 5; 7; 9; 11}? Да… Тут уже придётся задуматься более серьёзно. Ну конечно! Ведь это последнее, как говорят некоторые видеоблогеры, самое жёсткое задание в профильном ЕГЭ по математике. Так как же его решить?

Доводилось ли вам когда-нибудь слышать об осознанном переборе? Этот метод применяется тогда, когда возможных вариант не очень много. Но при этом варианты перебираются не как попало, а в определённой последовательности. Это нужно для того, чтобы не упустить из виду ни одного возможного варианта. Плюс, по возможности, при переборе исключаются из рассмотрения невозможные варианты. Итак, как же нам свести это задание к осознанному перебору?

Введём фильтр, ограничивающий перебор:

  • Заметим сразу, что суммы искомых хороших четырёхэлементных подмножеств должны быть чётными, иначе их нельзя разбить на подмножества с одинаковыми суммами элементами. При этом минимально возможная сумма равна 1+2+4+5 = 12, а максимально возможная сумма равна 5+7+9+11 = 32. Таких сумм 11 штук.
  • Примем также во внимание, что чётные числа 2 и 4 должны либо одновременно входить в хорошее четырёхэлементное множество, либо одновременно не входить в него. В противном случае только одно из чисел четырёхэлементного множества чётное, поэтому сумма элементов такого множества не будет чётной.
  • Поскольку порядок расположения элементов в искомых хороших четырёхэлементных множествах не важен, договоримся, что элементы в этих множествах будут у нас расположены по возрастанию.

Рассматриваем все возможные суммы:

  1. Сумма 12: {1; 2; 4; 5}.
  2. Сумма 14: {1; 2; 4; 7}.
  3. Сумма 16: нет вариантов.
  4. Сумма 18: {2; 4; 5; 7}.
  5. Сумма 20: нет вариантов.
  6. Сумма 22: {2; 4; 7; 9}, {2; 4; 5; 11}.
  7. Сумма 24: {1; 5; 7; 11}.
  8. Сумма 26: {2; 4; 9; 11}.
  9. Сумма 28: нет вариантов.
  10. Сумма 30: нет вариантов.
  11. Сумма 32: {5; 7; 9; 11}.

Вот и получилось у нас всего 8 множеств. Других вариантов нет. То есть ответ к заданию под буквой В — 8.

Вот такое решение задачи 19 из ЕГЭ по математике (профильный уровень). Для тех, кто только начинает готовиться к сдаче профильного ЕГЭ по математике, оно можно показаться сложным. Но на самом деле для решения таких задач требуется использование одних и тех же способов и приёмов. Нужно только овладеть ими, и все эти задачи будут казаться вам простыми, и вы их решите на экзамене без всяких проблем. Я вас мог этому научить. Подробную информацию обо мне и моих занятиях вы можете найти на .

Задание №19 ЕГЭ по математике весьма необычно. Для его решения необходимо применить знания в области теории чисел. Тем не менее, задание является весьма решаемым, однако для школьников с оценкой хорошо и ниже я рекомендовал бы оставить это задание на последнюю очередь. Перейдем к рассмотрению типового варианта.

Разбор типовых вариантов заданий №19 ЕГЭ по математике базового уровня

Вариант 19МБ1

Найдите трехзначное число, сумма цифр которого равна 20, а сумма квадратов цифр делится на 3, но не делится на 9. В ответе укажите какое-нибудь оно такое число.

Алгоритм выполнения:
  1. Ввести условные обозначения.
  2. Записать условия с помощью условных обозначений.
  3. Преобразовать полученные выражения.
  4. Логически рассуждая перебрать все возможные варианты, проверить их соответствие условиям.
Решение:

Обозначим первую цифру числа x, а вторую – y. Тогда третье число с учетом суммы цифр равной 20 будет равно 20 – (x + y). (x + y) обязательно меньше 10, иначе сумма равная 20 не получится.

По условию сумма квадратов цифр делится на 3, но не делится на 9. Запишем сумму квадратов цифр:

x 2 + y 2 + (20 – (x + y)) 2

Преобразуем полученное выражение. Преобразуем квадрат разности с учетом формулы приведения.

Квадрат разности двух выражений равен сумме квадратов этих выражений минус удвоенное произведение первого и второго выражений.

(20 – (x + y)) 2 = 400 -40(x + y) + (x + y) 2

Подставим получившееся выражение в начальное, получим:

x 2 + y 2 + (20 – (x + y)) 2 = x 2 + y 2 + 400 — 40(x + y) + (x + y) 2

Квадрат суммы двух выражений равен сумме квадратов этих выражений плюс удвоенное произведение первого и второго выражений.

(x + y) 2 = x 2 + 2xy + y 2

Подставим:

x 2 + y 2 + (20 – (x + y)) 2 = x 2 + y 2 + 400 — 40(x + y) + (x + y) 2 = x 2 + y 2 + 400 — 40(x + y) + x 2 + 2xy + y 2

Приведем подобные слагаемые(сложим x 2 с x 2 и y 2 с y 2), получим:

x 2 + y 2 + 400 — 40(x + y) + x 2 + 2xy + y 2 = 2x 2 + 2y 2 + 2 · 200 — 2 · 20(x + y) + 2xy

Вынесем множитель 2 за скобку:

2x 2 + 2y 2 + 2 · 200 — 2 · 20(x + y) + 2xy = 2(x 2 + y 2 + 200 — 20(x + y) + xy)

Для удобства объединим 200 и 20(x + y) и вынесем 20 за скобку, получим:

2(x 2 + y 2 + 20(10 — (x + y)) + xy)

Множитель 2 – четный, поэтому он никак не влияет на делимость на 3 или 9. Можем его не брать в расчет и рассматривать выражение:

x 2 + y 2 + 20(10 — (x + y)) + xy

Предположим, что и x, и y делятся на 3. Тогда x 2 + y 2 + xy делится на 3, а 20(10 — (x + y)) – не делится. Следовательно, и вся сумма x 2 + y 2 + 20(10 — (x + y)) + xy на 3 не делится.

Предположим, что на 3 делится только одна цифра. Тогда, учитывая, что (x + y) обязательно меньше 10, иначе сумма равная 20 не получится, подберем возможные пары.

(3;8), (6;5), (6;7), (6;8), (9;2), (9;4), (9;5), (9;7), (9;8).

Методом подстановки проверим, соответствуют эти пары условию.

x 2 + y 2 + 20(10 — (x + y)) + xy = 3 2 + 8 2 + 20(10 — (3 + 8)) + 3 · 8 = 9 + 64 – 20 + 24 = 77

x 2 + y 2 + 20(10 — (x + y)) + xy = 6 2 + 5 2 + 20(10 — (6 + 5)) + 6 · 5 = 36 + 25 – 20 + 30 = 71

x 2 + y 2 + 20(10 — (x + y)) + xy = 6 2 + 7 2 + 20(10 — (6 + 7)) + 6 · 7 = 36 + 49 – 60 + 42 = 67

x 2 + y 2 + 20(10 — (x + y)) + xy = 6 2 + 8 2 + 20(10 — (6 + 8)) + 6 · 8 = 36 + 64 – 80 + 48 = 68

x 2 + y 2 + 20(10 — (x + y)) + xy = 9 2 + 2 2 + 20(10 — (9 + 2)) + 9 · 2 = 81 + 4 – 20 + 18 = 83

x 2 + y 2 + 20(10 — (x + y)) + xy = 9 2 + 4 2 + 20(10 — (9 + 4)) + 9 · 4 = 81 + 16 – 60 + 36 = 73

Ни одна из полученных сумм не удовлетворяет условию «сумма квадратов цифр делится на 3, но не делится на 9».

Следующие пары можно не проверять, так как они дают уже имеющиеся тройки цифр.

Предположим, что ни одна из цифр числа не делится на 3.

Возможные пары:

(4;7), (5;7), (5;8), (7;8).

Проверим:

x 2 + y 2 + 20(10 — (x + y)) + xy = 4 2 + 7 2 + 20(10 — (4 + 7)) + 4 · 7 = 16 + 49 – 20 + 28 = 73

x 2 + y 2 + 20(10 — (x + y)) + xy = 5 2 + 7 2 + 20(10 — (5 + 7)) + 5 · 7 = 25 + 49 – 40 + 35 = 69

Сумма 69 удовлетворяет условию «сумма квадратов цифр делится на 3, но не делится на 9». Следовательно, подходят цифры 5,7,8 в любом порядке.

Вариант 19МБ2

На 6 карточках написаны цифры 1; 2; 3; 6; 9; 9 (по одной цифре на каждой карточке). В выражении □ + □□ + □□□ вместо каждого квадратика положили карточку из набора. Оказалось, что полученная сумма делится на 10. Найдите эту сумму. В ответе укажите какое-нибудь одно такое число.

Алгоритм выполнения:
  1. Вспомнить признак делимости на 10.
Решение:

1. Если сумма делится на 10 нацело, то последняя цифра должна быть 0, остальные цифры значения не имеют.

2. В первый квадрат поместим цифру 1, в следующем числе на последнем месте – цифру 3 (или 6), а в третьем – цифру 6 (или 3), получим (сумма 1+3+6=10):

3. Остальные цифры заполним произвольно, например, так:

и получится сумма

1+23+996 = 1020.

Ответ: 1020

Вариант 19МБ3

На 6 карточках написаны цифры 1; 2; 2; 3; 5; 7 (по одной цифре на каждой карточке). В выражении □ + □□ + □□□ вместо каждого квадратика положили карточку из набора. Оказалось, что полученная сумма делится на 20. Найдите эту сумму. В ответе укажите какое-нибудь одно такое число.

Алгоритм выполнения:
  1. Вспомнить признак делимости на 10 и сформулировать признак делимости на 20.
  2. Разместить последние цифры каждого слагаемого таким образом, чтобы в сумме получилось 10.
  3. Разместить предпоследние цифры каждого слагаемого таким образом, чтобы в сумме получилось четное число в результате с учетом суммы первых цифр.
  4. Расположить оставшиеся карточки в произвольном порядке.
Решение:

1. Чтобы сумма делилась на 20, она должна заканчиваться на 0 и вторая цифра с конца должна быть четной (делиться на 2). Чтобы в конце суммы получить 0, первые три карточки следует выбрать так:

2. Чтобы вторую цифру получить четной, можно взять карточки 2 и 7 (к ней будет добавляться еще 1 от первой суммы 10):

3. В последнее место помещаем оставшуюся цифру 1, в результате имеем:

и сумма равна:

Вариант 19МБ4

Найдите четырехзначное число, кратное 15, произведение цифр которого больше 0, но меньше 25. В ответе укажите какое-нибудь одно такое число.

Алгоритм выполнения
  1. Если произведение >0, то, значит, оно не равно нулю. Следовательно, ни один из множителей не может быть равным 0.
  2. Если произведение кратно 15, следовательно, оно кратно 5 и кратно 3.
  3. Если произведение кратно 5, то результат его должен оканчиваться 0 или 5. В данном случае берем 5, т.к. 0 не может быть одним из множителей (см.п.1).
  4. Итак, последняя цифра числа равна 5. Тогда произведение первых трех равно 25:5=5. Это означает, что нужно подобать 3 цифры так, чтобы их произведение было менее 5.
  5. Из всех полученных наборов цифр выбираем такой, чтобы сумма этих цифр плюс 5 (последняя, 4-я цифра) была кратной 3.
Решение:

Поскольку по условию произведение всех цифр кратно 15, то оно кратно 5 и 3.

Кратность 5 означает, что последней цифрой числа может быть только 0 или 5. Но 0 в виде последней цифры означал бы, что произведение всех 4-х цифр стало бы равным 0; а это противоречит условию. Тогда последняя цифра искомого числа равна 5.

Тогда получим: x·y·z·5

Меньше 5 произведение таких цифр: 1 1 1, 1 1 3, 1 1 2, 1 2 2.

Согласно признаку делимости на 3, выбираем из этих наборов такой, чтобы сумма его цифр плюс 5 делилась на 3:

1+1+1+5=8 – не подходит;

1+1+3+5=10 – не подходит;

1+2+2+5=10 – не подходит

1+1+2+5=9 – подходит.

Тогда условию задачи соответствуют числа: 1125 , 1215 , 2115 .

Ответ: 1125, 1215, 2115

Вариант 19МБ5

Вычеркните в числе 85417627 три цифры так, чтобы получившееся число делилось на 18. В ответе укажите какое-нибудь одно получившееся число.

Алгоритм выполнения
  1. Число делится на 18, если оно кратно 2 и 9.
  2. Кратность 2 означает, что число должно быть четным. Поэтому сразу отбрасывают последнюю – нечетную – цифру 7.
  3. Кратность 9 означает, что сумма его цифр делится на 9. Значит, находим сумму оставшихся цифр. Далее определяем подходящее для полученной суммы число, кратное 9. Число должно быть таким, чтобы: а) оно было меньшим суммы цифр; б) разница между этой суммой и найденным числом позволяла выделить в числе 2 цифры, сумма которых была бы равной этой разнице. Вычеркиваем эти цифры.
Решение:

Т.к. по условию число кратно 18, то оно кратно 2 и кратно 9.

Поскольку число кратно 2, то оно должно оканчиваться четной цифрой. 7 – нечетная цифра, поэтому вычеркиваем ее. Осталось: 8541762.

Т.к. полученное число кратно 9, то сумма его цифр должна делиться на 9. Находим общую сумму его цифр: 8+5+4+1+7+6+2=33. Ближайшее число, которое делится на 9, – это 27.

33–27=6 – это сумма двух цифр, которые нужно вычеркнуть. Пары цифр, которые при этом в сумме дают 6, – это 5 и 1 или 4 и 2. Вычеркнув их, получаем соответственно: 84762 или 85176 .

Кроме этого, на 9 делится 18. Тогда 33–18=15. В этом случае вычеркнуть придется 8 и 7. Получаем: 54162 .

На 9 делится еще и 9, однако 33–9=24, а пары цифр, которые дали бы в сумме 24, естественно, не существует.

Ответ: 84762, 85176, 54162

Вариант 19МБ6

На шести карточках написаны цифры 3; 6; 7; 7; 8; 9 (по одной цифре на каждой карточке). В выражении

Вместо каждого квадратика положили карточку из данного набора. Оказалось, что полученная сумма делится на 10, но не делится на 20.

В ответе укажите какую-нибудь одну такую сумму.

Алгоритм выполнения
  1. Во 2-м предложении текста задачи фактически представлено условие, при котором сумма делится на 10, однако не делится на 2.
  2. Из п.1 следует, что результирующее число должно оканчиваться 0, а предпоследняя его цифра должна быть нечетной.
Решение:

Для удобства восприятия разместим карточки в столбик:

Если число делится на 10, но не делится на 20, значит, оно точно не делится на 2 без последнего нуля.

Поскольку число кратно 10, то оно должно оканчиваться нулем. Поэтому в последнем разряде (единиц) нужно расположить 3 карточки с такими цифрами, чтоб их сумма оканчивалась на 0. Подходят здесь карточки: 1) 6, 7, 7; 2) 3, 8, 9. Их суммы равны 20. Соответственно, 0 мы пишем под чертой, а 2 переносим на предыдущий разряд (десятков):

Чтобы число не делилось на 20, необходимо, чтобы перед нулем стояла нечетная цифра. Нечетная сумма здесь получится тогда, когда одно из слагаемых будет нечетным, а два других четными. Одно из этих (других) слагаемых – это перенесенная 2. Поэтому из оставшихся цифр следует взять: 1) 3 и 8; 2) 6 и 7. Получаем:

На место сотен ставим последнюю (оставшуюся) карточку с цифрой: 1) 9; 2) 7. Получаем, соответственно, числа 1030 и 850 :

Ответ: 1030,850

Вариант 19МБ7

Найдите четное трехзначное на туральное число, сумма цифр которого на 1 меньше их произведения. В ответе укажите какое-нибудь одно такое число.

Алгоритм выполнения
  1. Вводим буквенные обозначения для цифр искомого числа. Исходя из условия задачи, составляем уравнение.
  2. Выражаем одну из цифр через 2 другие.
  3. Подбираем для этих 2-х (других) цифр значения так, чтобы 3-я (выраженная) представляло бы собой натуральное число. Вычисляем 3-ю цифру.
  4. Формируем искомое число так, чтобы оно было четным.
Решение:

Пусть цифры искомого числа – x, y, z. Тогда получаем:

xyz–x–y–z=1

z=(x+y+1)/(xy–1)

Знаменатель в этом выражении должен быть целым и положительным. Для простоты (а также для гарантии правильных расчетов) примем, что он должен быть равен 1. Тогда имеем: ху–1=1 → ху=2. Поскольку х и у это цифры, то их значения могут быть равными только 1 и 2 (т.к. только произведение этих однозначных натур.чисел дает в результате 2).

Отсюда z составляет: z=(1+2+1)/(1·2–1)=4/1=4.

Итак, имеем цифры: 1, 2, 4.

Т.к. по условию итоговое число должно быть четным, то оканчиваться оно может только 2 или 4. Тогда правильными вариантами чисел будут такие:

124 , 142 , 214 , 412 .

Ответ: 124, 142, 214, 412

Вариант 19МБ8

Найдите шестизначное число, которое записывается только цифрами 2 и 0 и делится на 24. В ответе укажите какое-нибудь одно такое число.

Алгоритм выполнения
  1. Если число делится на 24, значит, оно делится на 8 и на 3.
  2. Согласно признаку делимости на 8, 3 последних цифры его должны образовывать число, которое кратно 8.
  3. Чтобы число делилось на 3, необходимо, чтобы сумма его цифр делилась на 3. Учитывая уже сформированную 2-ю часть числа (см.п.2), дополняем его первыми тремя цифрами соответственно.
Решение:

Чтобы искомое число было кратно 24, требуется, чтобы оно делилось на 8 и в то же время на 3.

Число делится на 8, если последние его 3 цифры образуют число, кратное 8. С использованием только двоек и нулей такое трехзначное число можно образовать так: 000, 002, 020, 022, 200, 202, 220, 222. Из этих чисел на 8 делится только 000 и 200.

Теперь нужно дополнить искомое число первыми 3-мя цифрами так, чтобы оно делилось еще и на 3.

В 1-м случае это будет единственный вариант: 222000 .

Во 2-м случае вариантов два: 220200 , 202200 .

Ответ: 222000, 220200, 202200

Вариант 19МБ9

Найдите четырехзначное число, кратное 15, произведение цифр которого больше 35, но меньше 45. В ответе укажите какое-нибудь одно такое число.

Алгоритм выполнения
  1. Если число кратно 15, значит, оно кратно 3 и 5.
  2. Применяем признак делимости на 5 и условие задачи, согласно которому произведение цифр числа ≠0. Так получаем, что последняя цифра искомого числа – только 5.
  3. Делим 35 на 5 и 45 на 5. Узнаем диапазон значений, которые может принимать произведение первых 3-х цифр числа. Узнаем, что оно может быть равно только 8.
  4. Определяем последовательности цифр, которые дают при перемножении 8.
  5. Проверяем полученные из найденных цифр числа на кратность трем.
Решение:

Кратность искомого числа 15 дает 2 условия: оно должно делиться на 5 и на 3.

Если число кратно 5, то оно должно оканчиваться цифрой 5 или 0. Однако 0 в данном случае использовать нельзя, поскольку при этом произведение цифр числа оказывается равным 0. По условию же это не так. Итак, последняя – 4-я – цифра числа равна 5.

По условию 35

1·1·8=8, 1·2·4=8.

Отсюда получаем числа:

1185 ; 1245 .

Проверяем их на кратность 3:

Вывод: оба найденные числа кратны 3. Плюс кратны их комбинации:

1815 ; 8115 ; 1425 ; 2145 ; 2415 ; 4125 ; 4215 .

Ответ: 1815; 8115; 1425; 2145; 2415; 4125; 4215

Вариант 19МБ10

Найдите пятизначные число, кратное 25, любые две соседние цифры которого отличаются на 2. В ответе укажите какое-нибудь одно такое число.

Алгоритм выполнения
  1. Принимаем во внимание, что на 25 делятся числа, которые придется последовательно делить на 5 дважды. Определяем, какой парой цифр они должны оканчиваться.
  2. Учитывая, что 2-й частью условия является различие каждой соседней пары цифр исключительно на 2 единицы, выбираем подходящий вариант (или варианты) цифр.
  3. Способом подбора находим остальные цифры и, соответственно, числа. Одно из них запишем в ответе.
Решение:

Если число делится на 25, то оно должно оканчиваться на: 00, 25, 50, 75. Т.к. соседние цифры должны отличаться строго на 2, то использовать для 4-й и 5-й цифр можем только 75. Получаем: ***75.

  1. **975 или
  2. **575.

1) *7975 → 97975 или 57975 ;

2) *3575 → 13575 или 53575 , *7575 → 57575 или 97575 .

Ответ: 97975, 57975, 13575, 53575, 57575, 97575

Вариант 19МБ11

Найдите трехзначное натуральное число, большее 600, которое при делении на 3, на 4 и на 5 дает в остатке 1 и цифры которого расположены в порядке убывания слева направо. В ответе укажите какое-нибудь такое число.

Алгоритм выполнения
  1. Определяем диапазон значений для 1-й цифры числа (сотен).
  2. Определяем, какой может быть последняя цифра (единицы), приняв во внимание: 1) при делении на 5 дает в остатке 1; 2) на этом месте не может быть четная цифра, поскольку это одно из условий делимости на 4.
  3. Способом подбора определяем набор чисел, которые при делении на 3 дают в остатке 1.
  4. Из этого набора (см.п.3) отбрасываем числа, которые при делении на 4 дают остаток, отличный от 1.
Решение:

Т.к. искомое число >600 и при этом является трехзначным, то 1-й цифрой может быть только 6, 7, 8 или 9. Тогда получаем для искомого числа:

Если число при делении на 5 должно давать в остатке 1, значит, оно может оканчиваться только на 0+1=1 или на 5+1=6. Шестерку тут отбрасываем, поскольку в этом случае число четное и потенциально может делиться на 4. Поэтому имеем:

Если число при делении на 3 дает в остатке 1, значит, сумма его цифр должна быть кратной 3 плюс 1. Кроме того, учитываем, что цифры должны располагаться в числе в порядке убывания. Подбираем такие числа:

Из этой последовательности отбрасываем числа, для которых не выполняется условие о том, что число при делении на 4 должно давать в остатке 1.

Т.к. признак делимости на 4 заключается в том, что 2 последние цифры должны делиться на 4, то получаем:

для 631: 31=28+3, т.е. в остатке имеем 3; число не подходит

для 721 : 21=20+1, т.е. в остатке – 1; число подходит

для 751: 51=48+3, т.е. в остатке – 3; число не подходит

для 841 : 41=40+1, т.е. в остатке – 1; число подходит

для 871: 71=68+3, т.е. в остатке – 3; число не подходит

для 931: 31=28+3, т.е. в остатке – 3; число не подходит

для 961 : 61=60+1, т.е. в остатке – 1; число подходит

Ответ: 721, 841, 961

Вариант 19МБ12

Найдите трехзначное натуральное число, большее 400, но меньшее 650, которое делится на каждую свою цифру и все цифры которого различны и не равны 0. В ответе укажите какое-нибудь одно такое число.

Алгоритм выполнения
  1. Из условия следует, что числа могут начинаться только на 4,5 или 6.
  2. При анализе чисел 4-й сотни отбрасываем числа: 1) 1-го десятка, т.к. в них содержится 0; 2) 4-го десятка, т.к. в этом случае первые две цифры совпадут; 3) числа 5-го десятка, т.к. они должны оканчиваться только на 5 или 0, что недопустимо. Кроме того, для всех четных десятков можно рассматривать только четные числа.
  3. Числа 5-й сотни отбрасываем полностью, т.к. чтобы делиться на каждую свою цифру, они должны оканчиваться 5 или 0.
  4. Для чисел 6-й сотни рассматривать можно только: 1) четные; 2) кратные 3; 3) не оканчивающиеся 0.
Решение:

Числа 40* и 4*0 отбрасываем, т.к. они содержат 0.

Числа 41* годятся только четные, т.к. это обязательное условия для кратности 4. Анализируем:

412 – подходит

414 – не подходит, т.к. в нем совпадают цифры

416 – не подходит, т.к. не делится на 6

418 – не подходит, т.к. не делится ни на 4, ни на 8

Из чисел 42* годятся только четные, поскольку должны делиться на 2:

422 и 424 – не подходят, т.к. в них совпадают цифры

426 – не подходит, т.к. не делится на 4

428 – не подходит, т.к. не делится на 8

Числа 43* годятся только четные и кратные 3. Поэтому тут подходит только 432 .

Числа 44* не подходят полностью.

Числа 45* не подходят полностью, т.к. они должны оканчиваться только 5 (т.е. быть нечетными) или 0.

Числа 46*, 47*, 48*, 49* не подходят полностью, т.к. для каждого из них не выполняется 1 или несколько условий.

Числа 5-й сотни не годятся полностью. Они должны делиться на 5, а для этого оканчиваться либо 5, либо 0, что не допускается.

Числа 60* не годятся полностью.

Среди остальных можно рассматривать только четные, кратные 3, не оканчивающиеся 0. Опуская подробности перебора чисел, оговорим только, что из них годятся: 612 , 624 , 648 . Для остальных не выполняется одно или несколько условий.

Ответ: 412, 432, 612, 624, 648

Вариант 19МБ13

Найдите четырехзначное число, кратное 45, все цифры которого различны и четны. В ответе укажите какое-нибудь одно такое число.

Алгоритм выполнения
  1. Если число кратно 45, значит, оно делится на 5 и на 9.
  2. Рассматривать следует только числа четных сотен.
  3. Оканчиваться числа могут только 0, т.к. 5 – нечетная цифра.
  4. Сумма цифр числа должна быть равна 18. Только в этом случае можно составить его из всех четных цифр.
Решение:

Т.к. по условию цифры должны быть четными, то рассматривать можно только числа 2-й, 4-й, 6-й и 8-й тысяч. Это значит, что начинаться оно может с 2, 4, 6 или 8.

Если число кратно 45, то оно кратно 5 и кратно 9.

Если число кратно 5, то оно должно оканчиваться 5 или 0. Но поскольку все цифры должны быть четными, то подходит здесь только 0.

Т.о., получаем шаблоны чисел: 2**0, 4**0, 6**0, 8**0. Отсюда следует, что для проверки кратности 9 требуется, чтобы сумма первых 3-х цифр была равной 9, или 18, или 27 и т.д. Но подходит тут только 18. Основания: 1) для получения в сумме 9 нужно, чтобы одно из слагаемых было нечетным, а это противоречит условию; 2) 27 не подходит потому, что даже если взять самую большую 1-ю цифру 8, то сумма 2-й и 3-й цифр будет равна 27–8=19, что превышает допустимый предел. Еще большие суммы цифр, кратные 9, не подходят тем более.

Рассматриваем числа по тысячам.

Числа 2**0. Сумма средних цифр равна: 18–2=16. Получить 16 из четных чисел можно только так: 8+8. Однако цифры не должны повторяться. Поэтому подходящих условию чисел здесь нет.

Числа 4**0. Сумма средних цифр: 18–4=14. 14=8+6. Поэтому получаем: 4680 или 4860 .

Числа 6**0. Сумма средних цифр: 18–6=12. 12=6+6, что не подходит, т.к. цифры повторяются. 12=4+8. Получаем: 6480 или 6840 .

Числа 8**0. Сумма средних цифр: 18–8=10. 10=2+8, что не подходит, т.к. при этом будет повторяться 8. 10=4+6. Получаем: 8460 или 8640 .

Ответ: 4680, 4860, 6480, 6840, 8460, 8640

Приведите пример трёхзначного числа, сумма цифр которого равна 20, а сумма квадратов цифр делится на 3, но не делится на 9.

Решение.

Разложим число 20 на слагаемые различными способами:

20 = 9 + 9 + 2 = 9 + 8 + 3 = 9 + 7 + 4 = 9 + 6 + 5 = 8 + 8 + 4 = 8 + 7 + 5 = 8 + 6 + 6 = 7 + 7 + 6.

При разложении способами 1−4, 7 и 8 суммы квадратов чисел не кратны трём. При разложении пятым способом сумма квадратов кратна девяти. Разложение шестым способом удовлетворяет условиям задачи. Таким образом, условию задачи удовлетворяет любое число, записанное цифрами 5, 7 и 8, например, число 578.

Ответ: 578|587|758|785|857|875

Источник: Де­мон­стра­ци­он­ная вер­сия ЕГЭ — 2015.

Найдите трёхзначное натуральное число, большее 400, которое при делении на 6 и на 5 даёт равные ненулевые остатки и первая слева цифра которого является средним арифметическим двух других цифр. В ответе укажите какое-нибудь одно такое число.

Решение.

Число имеет одинаковые остатки при делении на 5 и на 6, следовательно, число имеет тот же остаток при делении на 30, причём этот остаток не равен нулю и меньше пяти. Таким образом, искомое число может иметь вид: .

При . Ни одно из чисел не больше 400

При : 421, 422, 423, 424. Первая слева цифра не является средним арифметическим двух других цифр

При : 451, 452, 453, 454. Число 453 удовлетворяет всем условиям задачи.

Также подходят числа 573 и 693.

Ответ: 453,573, 693.

Ответ: 453|573|693

Найдите четырёхзначное число, кратное 22, произведение цифр которого равно 24. В ответе укажите какое-нибудь одно такое число.

Решение.

Чтобы число abcd делилось на 22, оно должно делиться и на 2, и на 11. Произведение цифр 24 можно представить многими способами, основой которых являются произведения — . Признак делимости на 11: Число делится на 11, если сумма цифр, которые стоят на четных местах равна сумме цифр, стоящих на нечетных местах, либо отличается от неё на 11. Таким образом, a+c=b+d или a+c=b+d+11 или a+c+11=b+d. Кроме того, раз число делится на 2, то оно должно быть четным. Согласно перечисленным признакам можно подобрать следующие числа: 4312, 2134, 1342, 3124

Ответ: 2134|4312|1342|3124

Найдите трёхзначное число, кратное 25, все цифры которого различны, а сумма квадратов цифр делится на 3, но не делится на 9. В ответе укажите какое-нибудь одно такое число.

Решение.

Чтобы число делилось на 25, оно должно заканчиваться на 00, 25, 50 или 75. Наше число на 00 заканчиваться не может, поскольку все его цифры должны быть различны. Выпишем все трёхзначные числа, заканчивающиеся на 25, 50 или 75, все цифры которых различны, найдём сумму квадратов их цифр, проверим, делится ли она на 3 и на 9.

Сумма цифр не делится на 3.

Сумма цифр делится на 3, но не делится на 9. Это искомое число.

Сумма цифр не делится на 3.

Сумма цифр делится на 3, но не делится на 9. Это искомое число.

Сумма цифр не делится на 3.

Сумма цифр не делится на 3.

Сумма цифр не делится на 3.

Сумма цифр не делится на 3.

Сумма цифр делится на 3 и на 9.

Сумма цифр не делится на 3.

Сумма цифр не делится на 3.

Сумма цифр не делится на 3.

Сумма цифр делится на 3, но не делится на 9. Это искомое число.

Сумма цифр не делится на 3.

Сумма цифр делится на 3, но не делится на 9. Это искомое число.

Сумма цифр не делится на 3.

Сумма цифр делится на 3, но не делится на 9. Это искомое число.

Сумма цифр не делится на 3.

Сумма цифр не делится на 3.

Сумма цифр не делится на 3.

Задание №19 из базового ЕГЭ по математикеmathvideourok.moy.su

Признаки делимости на 2 и 4:

Число делится на 2, если оно заканчивается четной
цифрой или нулём.
Числа 2346 и 3650 — делятся на 2. Число 4521 — не
делится на 2.
Число делится на 4, если две последние его
цифры нули или образуют число, делящееся на 4. В

Числа 31700 и 16608 -делятся на 4. 215634 – не
делится на 4.

Признаки делимости на 3 и 9:

На 3 делятся только те числа, у которых сумма
цифр делится на 3.
Числа 17835 и 5472 – делятся на 3. Число 105499 – не
делится на 3.
На 9 делятся только те числа, у которых сумма
цифр делится на 9.
Числа 2376 и 342000 – делятся на 9. Число 106499 – не
делится на 9.

Признаки делимости на 8 и 6:

Число делится на 8, если три последние цифры его
нули или образуют число, делящееся на 8. В
остальных случаях — не делится.
Числа 125000 и 111120 – делятся на 8. Числа 170004 и
124300 – не делятся на 8.
Число делится на 6, если оно делится одновременно
на 2 и на 3. В противном случае — не делится.
Числа 126 и 254610 – делятся на 6. Числа 3585 и 6574 не делятся на 6.

Признаки делимости на 5 и 25:

На 5 делятся числа, последняя цифра которых 0
или 5. Другие — не делятся.
Числа 245 и 56780 – делятся на 5. Числа 451 и 678 – не
делятся на 5.
На 25 делятся числа, две последние цифры которых
нули или образуют число, делящееся на 25 (т. е.
числа, оканчивающиеся на 00, 25, 50 или 75). Другие
не делятся.
Числа 7150 и 345600 – делятся на 25. Число 56755 – не
делится на 25.

Признаки делимости на 10, 100 и 1000:

На 10 делятся только те числа, последняя цифра
которых нуль, на 100 — только те числа, у которых
две последние цифры нули, на 1000 — только те, у
которых три последние цифры нули.
Число 34680 – делится на 10. Число 56700 – делится на
100 и на 10. Число 87549000 — делится на 10, 100 и 1000.
Числа 75864, 7776539 и 9864032 – не делятся на 10, 100 и
1000.

Признак делимости на 11:

На 11 делятся только те числа, у которых сумма цифр,
занимающих нечетные места, либо равна сумме цифр,
занимающих четные места, либо разнится от нее на число,
делящееся на 11.
Число 103785 делится на 11, так как сумма цифр, занимающих
нечетные места, 1+3+8=12 равна сумме цифр, занимающих четные
места 0+7+5=12.
Число 9163627 делится на 11, так как сумма цифр, занимающих
нечетные места, есть 9 + 6 + 6 + 7 = 28, а сумма цифр, занимающих
четные места, есть 1 + 3 +2 =6; разность между числами 28 и 6 есть
22, а это число делится на 11.
Число 461025 не делится на 11, так как числа 4+ 1 + 2 = 7 и б +0 +
5=11 не равны друг другу, а их разность 11 -7 = 4 на 11 не делится.

Описание презентации по отдельным слайдам:

1 слайд

Описание слайда:

2 слайд

Описание слайда:

Приведите пример трёхзначного числа, сумма цифр которого равна 20, а сумма квадратов цифр делится на 3, но не делится на 9. Разложим число 20 на слагаемые различными способами: 1) 20 = 9 + 9 + 2 2) 20 = 9 + 8 + 3 3) 20 = 9 + 7 + 4 4) 20 = 9 + 6 + 5 5) 20 = 8 + 8 + 4 6) 20 = 8 + 7 + 5. Находим сумму квадратов в каждом разложении и проверяем, делится ли она на 3 и не делится на 9. При разложении способами (1)−(4) суммы квадратов чисел не делятся на 3. При разложении способом (5) сумма квадратов делится на 3 и на 9. Разложение способом (6) удовлетворяет условиям задачи. Ответ: например, числа 578 или 587 или 785 и т.д.

3 слайд

Описание слайда:

№ 2. Приведите пример трехзначного натурального числа, большего 600, которое при делении на 3, на 4 и на 5 даёт в остатке 1 и цифры которого расположены в порядке убывания слева направо. В ответе укажите ровно одно такое число. 600 делится на 3, 4 и 5. Число 601 дает в остатке 1 при делении на эти числа, но цифры в 601 не убывают. НОК=3*4*5=60 — делится на 3, 4 и 5. Проверяем число 600+60 =660. Оно делится на 3, 4 и 5, число с остатком 1 это 661, но цифры не убывают. Проверяем следующее 660+60= 720, оно делится на 3, 4 и 5. Число 721 дает в остатке 1 и цифры убывают. Ответ: 721.

4 слайд

Описание слайда:

№ 3. Приведите пример пятизначного числа, кратного 12, произведение цифр которого равно 40. В ответе укажите ровно одно такое число. Разложим 40 на 5 множителей: 40=5*2*2*2*1. Например, 51222. Т.к. число должно быть кратно 12, то оно должно делиться на 3 и 4. Сумма цифр равна 12, значит, оно делится на 3. Чтобы число делилось на 4, надо чтобы две последние цифры составили число, которое делится на 4. 22 не делится на 4, а 12 делится. Значит, в конце стоят цифры 1, 2. Варианты ответа: 52212, 25212, 22512.

5 слайд

Описание слайда:

№ 4. Вычеркните в числе 53164018 три цифры так, чтобы получившееся число делилось на 15. В ответе укажите ровно одно получившееся число 5 3 1 6 4 0 1 8 — цифры числа. Чтобы число делилось на 15, надо, чтобы оно делилось на 3 и на 5. Чтобы число делилось на 5, надо, чтобы оно оканчивалось на 0 или на 5. Вычеркнем 2 последние цифры. 5+3+1+6+4+0 = 19, значит надо вычеркнуть цифру 1 (сумма цифр будет 18), или 4 (сумма цифр будет 15). Варианты ответа: 53640 или 53160.

6 слайд

Описание слайда:

№ 5. Найдите трехзначное число большее 500 которое при делении на 4 на 5 и на 6 дает в остатке 2 и в записи которого есть только две различные цифры. В ответе укажите какое-нибудь одно такое число. Число которое делится на 4, 5 и 6 равно 60. Число больше 500 и кратное 60 это 540, 600, 660, 720, 780, 840, 900, 960. Чтобы при делении на 60 в остатке получить 2, надо к любому из этих чисел прибавить 2. Это может быть 662 или 722.

7 слайд

№ 7. Найдите трехзначное натуральное число, большее 400, но меньшее 650, которое делится на каждую свою цифру и все цифры которого различны и не равны нулю. В ответе укажите какое-нибудь одно такое число. Число начинается с цифры 4 (больше 400), значит оно должно делиться на 4. Второе число — 416. Оно делится и на 4. но не делиться на 6. Первое число — 412. Оно делится и на 4 и на 2 (четное число) Число делится на 4, если оканчивается на 00, или число, составленное из двух последних цифр данного числа, делится на 4. Еще число — 432. Оно делится и на 4, и на 3, и на 2. Варианты ответа: 412 или 432.

Вопросы»Базовый уровень по математике с решениями. Задание 19. ЕГЭ |Поступи в ВУЗ

Базовый уровень по математике с решениями. Задание 19. ЕГЭ

создана: 17.11.2019 в 22:10
…………………………………………

liliana :

В задании 19 базового уровня предложены задачи на тему «Делимость натуральных чисел». Чтобы решить такую задачу, надо хорошо знать признаки делимости натуральных чисел.

Признаки делимости.

Признаки делимости на 2, 3, 4, 6, 8, 9, 11, 5, 25, 10, 100, 1000.

1. Признак делимости на 2. Число делится на 2, если его последняя цифра — ноль или делится на 2. Числа, делящиеся на два, называются чётными, не делящиеся на два – нечётными.

2. Признак делимости на 4. Число делится на 4, если две его последние цифры — нули или образуют число, которое делится на 4.

3. Признак делимости на 8. Число делится на 8, если три его последние цифры — нули или образуют число, которое делится на 8.

4. Признаки делимости на 3 и 9. Число делится на 3, если его сумма цифр делится на 3. Число делится на 9, если его сумма цифр делится на 9.

5. Признак делимости на 6. Число делится на 6, если оно делится на 2 и на 3.

6. Признак делимости на 5. Число делится на 5, если его последняя цифра — ноль или 5.

7. Признак делимости на 25. Число делится на 25, если две его последние цифры — нули или образуют число, которое делится на 25.

8. Признак делимости на 10. Число делится на 10, если его последняя цифра — ноль.

9. Признак делимости на 100. Число делится на 100, если две его последние цифры – нули.

10. Признак делимости на 1000. Число делится на 1000, если три его последние цифры -нули.

11. Признак делимости на 11. На 11 делятся только те числа, у которых сумма цифр, стоящих на нечётных местах, либо равна сумме цифр, стоящих на чётных местах, либо отличается от неё на число, делящееся на 11. (Например, 12364 делится на 11, т.к. 1+3+4=2+6.)

 

 

За­да­ние 19 (1). При­ве­ди­те при­мер трёхзнач­но­го числа, сумма цифр ко­то­ро­го равна 20, а сумма квад­ра­тов цифр де­лит­ся на 3, но не де­лит­ся на 9.

Решение.

Раз­ло­жим число 20 на сла­га­е­мые раз­лич­ны­ми спо­со­ба­ми:

1) 20 = 9 + 9 + 2

2) 20 = 9 + 8 + 3

3) 20 = 9 + 7 + 4

4) 20 = 9 + 6 + 5

5) 20 = 8 + 8 + 4

6) 20 = 8 + 7 + 5.

Находим сумму квадратов в каждом разложении и проверяем, делится ли она на 3 и не делится на 9?

Замечаем, что, если в разложении 2 числа делятся на 3, то сумма квадратов на 3 не делится.

92+92+22    не делится на 3

При раз­ло­же­нии спо­со­ба­ми (1)−(4) суммы квад­ра­тов чисел не делятся на 3.

При раз­ло­же­нии спо­со­бом (5) сумма квад­ра­тов делится на 3 и на 9.

Раз­ло­же­ние ше­стым спо­со­бом удо­вле­тво­ря­ет усло­ви­ям за­да­чи. Таким об­ра­зом, усло­вию за­да­чи удо­вле­тво­ря­ет любое число, за­пи­сан­ное циф­ра­ми 5, 7 и 8, на­при­мер, числа 578 или 587 или 785 и т.д.

Сумма и произведение цифр числа. Решение задачи на Python

Одной из часто используемых задач для начинающих изучать программирование является нахождение суммы и произведения цифр числа. Число может вводиться с клавиатуры или генерироваться случайное число. Задача формулируется так:

Дано число. Найти сумму и произведение его цифр.

Например, сумма цифр числа 253 равна 10-ти, так как 2 + 5 + 3 = 10. Произведение цифр числа 253 равно 30-ти, так как 2 * 5 * 3 = 30.

Обычно предполагается, что данная задача должна быть решена арифметическим способом. То есть с заданным число должны производиться определенные арифметические действия, позволяющие извлечь из него все цифры, затем сложить их и перемножить.

И здесь на помощь приходят операции деления нацело и нахождения остатка. Если число разделить нацело на 10, произойдет «потеря» последней цифры числа. Например, 253 ÷ 10 = 25 (остаток 3). С другой стороны, эта потерянная цифра есть остаток от деления. Получив эту цифру, мы можем добавить ее к сумме цифр и умножить на нее произведение цифр числа.

Пусть n – само число, suma – сумма его цифр, а mult – произведение. Тогда алгоритм нахождения суммы и произведения цифр можно словесно описать так:

  1. Переменной suma присвоить ноль.
  2. Переменной mult присвоить единицу. Присваивать 0 нельзя, так как при умножении на ноль результат будет нулевым.
  3. Пока значение переменной n больше нуля повторять следующие действия:
    1. Найти остаток от деления значения n на 10, то есть извлечь последнюю цифру числа.
    2. Добавить извлеченную цифру к сумме и увеличить на эту цифру произведение.
    3. Избавиться от последнего разряда числа n путем деления нацело на 10.

В языке Python операция нахождения остатка от деления обозначается знаком процента — %. Деление нацело — двумя слэшами — //.

Код программы на языке Python

n = int(input())

suma = 0
mult = 1

while n > 0:
    digit = n % 10
    suma = suma + digit
    mult = mult * digit
    n = n // 10

print("Сумма:", suma)
print("Произведение:", mult)

Пример выполнения:

253
Сумма: 10
Произведение: 30

Изменение значений переменных можно записать в сокращенном виде:

...
while n > 0:
    digit = n % 10
    suma += digit
    mult *= digit
    n //= 10
...

Приведенная выше программа подходит только для нахождения суммы и произведения цифр натуральных чисел, то есть целых чисел больше нуля. Если исходное число может быть любым целым, следует учесть обработку отрицательных чисел и нуля.

Если число отрицательное, это не влияет на сумму его цифр. В таком случае достаточно будет использовать встроенную в Python функции abc(), которая возвращает абсолютное значение переданного ей аргумента. Она превратит отрицательное число в положительное, и цикл while с его условием n > 0 будет работать как и прежде.

Если число равно нулю, то по логике вещей сумма его цифр и их произведение должны иметь нулевые значения. Цикл срабатывать не будет. Поскольку исходное значение mult — это 1, следует добавить проверку на случай, если заданное число — это ноль.

Программа, обрабатывающая все целые числа, может начинаться так:

n = abs(int(input()))

suma = 0
mult = 1
if n == 0:
    mult = 0
...

Заметим, если в самом числе встречается цифра 0 (например, 503), то произведение всех цифр будет равно нулю. Усложним задачу:

Вводится натуральное число. Найти сумму и произведение цифр, из которых состоит это число. При этом если в числе встречается цифра 0, то ее не надо учитывать при нахождении произведения.

Для решения такой задачи в цикл добавляется проверка извлеченной цифры на ее неравенство нулю. Делать это надо до умножения на нее значения переменной-произведения.

n = int(input())

suma = 0
mult = 1

while n > 0:
    digit = n % 10
    if digit != 0:  
        suma += digit
        mult *= digit
    n = n // 10

print("Сумма:", suma)
print("Произведение:", mult)

Обратим внимание, что заголовок условного оператора if digit != 0: в Python можно сократить до просто if digit:. Потому что 0 — это false. Все остальные числа считаются истиной.

Приведенный выше математический алгоритм нахождения суммы и произведения цифр числа можно назвать классическим, или универсальным. Подобным способом задачу можно решить на всех императивных языках, независимо от богатства их инструментария. Однако средства языка программирования могут позволить решить задачу другим, зачастую более простым, путем. Например, в Python можно не преобразовывать введенную строку с числу, а извлекать из нее отдельные символы, которые преобразовывать к целочисленному типу int:

a = input()

suma = 0
mult = 1

for digit in a:
    suma += int(digit)
    mult *= int(digit)

print("Сумма:", suma)
print("Произведение:", mult)

Если добавить в код проверку, что извлеченный символ строки действительно является цифрой, то программа станет более универсальной. С ее помощью можно будет считать не только сумму и произведение цифр целых чисел, но и вещественных, а также цифр, извлекаемых из произвольной строки.

n = input()

suma = 0
mult = 1

for digit in n:
    if digit.isdigit():
        suma += int(digit)
        mult *= int(digit)

print("Сумма:", suma)
print("Произведение:", mult)

Пример выполнения:

это3 чи3с9ло!
Сумма: 15
Произведение: 81

Строковый метод isdigit() проверяет, состоит ли строка только из цифр. В нашем случае роль строки играет одиночный, извлеченный на текущей итерации цикла, символ.

Глубокое знание языка Python позволяет решить задачу более экзотическими способами:

import functools

n = list(input())
n = [int(digit) for digit in n]

suma = sum(n)
mult = functools.reduce(lambda x, y: x*y, n)

print("Сумма:", suma)
print("Произведение:", mult)

Встроенная функция list() преобразует переданную ей строку в список. Так если заданная строка — "234", то получится список ['2', '3', '4'].

Выражение [int(digit) for digit in n] представляет собой генератор списка. В данном случае будет получен список чисел: [2, 3, 4].

Встроенная функция sum() считает сумму элементов переданного ей аргумента.

Функция reduce() модуля functools() принимает два аргумента — лямбда-выражение и в данном случае список. Здесь в переменной x происходит накопление произведения, а y принимает каждое следующее значение списка.

Текстовые задачи. Задачи на числовые зависимости

Задачи на числовые зависимости с решениями

перейти к содержанию курса текстовых задач

  1. Найти двузначное число, зная, что число его единиц на 2 больше числа десятков и что произведение искомого числа на сумму его цифр равно 144. Решение
  2. Ученику надо было умножить 78 на двузначное число, в котором цифра десятков втрое больше цифры единиц; по ошибке он переставил цифры во втором сомножителе, отчего и получил произведение, на 2808 меньшее истинного. Чему равно истинное произведение? Решение
  3. Сумма цифр двузначного числа равна 12. Если к этому числу прибавить 36, то получится число, записанное теми же цифрами, но в обратном порядке. Найти искомое двузначное число. Решение
  4. Остаток от деления натурального числаnна 12 равен 5; остаток от деления n на 16 равен 9. Чему равен остаток от деления наименьшего из возможных чисел n на 24?
  5. При каких натуральных число также будет натуральным? Решение
  6. При каких натуральных дробь сократима? Решение
  7. Сумма квадратов крайних чисел четырехзначного числа M равна 58. Сумма квадратов средних цифр этого числа равна 68. Сумма числа M и числа 4536 равна числу, записанному теми же цифрами числа М, но в обратном порядке. Найдите число M. Решение
  8. Запись шестизначного числа начинается цифрой 2. Если эту цифру перенести с первого места на последнее, сохранив порядок остальных пяти цифр, то новое число будет втрое больше первоначального. Найти первоначальное число.
  9. Докажите, что при любом натуральном
  10. Ученик должен был перемножить два трехзначных числа. Однако он не заметил знака умножения и принял оба рядом стоящих множителя за одно шестизначное число. Поэтому получившееся ошибочное число оказалось в 3 раза больше истинного произведения. Какие числа должен был перемножить ученик?
  11. Знаменатель несократимой дроби на 2 больше, чем числитель. Если у дроби, обратной данной, уменьшить числитель на 3 и вычесть из этой новой дроби заданную, то получится . Найти первоначальную дробь.

Задачи для самостоятельного решения

  1. Сумма двух чисел, умноженная на сумму квадратов этих чисел, равна 369, а разность их, умноженная на разность их квадратов, равна 9. Найдите числа. Ответ: 5 и 4
  2. Число 128 представлено в виде суммы четырех слагаемых так, что первое слагаемое относится ко второму как 2:3, второе к третьему — как 3:5, а третье к четвертому — как 5:6. Найдите эти слагаемые. Ответ: 16, 24, 40, 48
  3. Числитель несократимой дроби на 1 меньше, чем ее знаменатель. Если умножить числитель на 6, а знаменатель увеличить на 19, то получится дробь, обратная исходной. Найдите произведение числителя и знаменателя исходной дроби. Ответ: 36
  4. После деления некоторого двузначного числа на сумму его цифр в частном получается 7, а в остатке 6. После деления этого числа на произведение его цифр в частном получается 3, а в остатке 11. Найти это число. Ответ:  83
  5. Если двузначное число разделить на сумму его цифр, то в частном получится 3 и в остатке получится 3. Найдите это число, если разность квадратов его цифр по модулю в 2 раза больше квадрата разности его цифр. Ответ: 39
  6. Первое число при делении на второе дает в частном 2 и в остатке 3. Второе при делении на третье дает в частном 1  и в остатке 8. Третье число при делении на четвертое дает в частном 2 и в остатке 1. Найдите эти четыре числа, если их сумма равна 76. Ответ: 41, 19, 11, 5
  7. Сумма двух чисел равна 17, а сумма их кубов равна 1547. Найдите большее из этих чисел. Ответ: 11
  8. Сумма двух чисел равна 495. Одно из них оканчивается нулем. Если его убрать, то получится второе число. Найдите эти числа. Ответ: 450, 45
  9. Задумано натуральное число. К его записи присоединили справа цифру 7 и из полученного нового числа вычли квадрат задуманного числа. Результат уменьшили на 75% и еще раз вычли задуманное число. В итого получился нуль. Найдите задуманное числа. Ответ: 7
  10.  Первая цифра трехзначного числа равна 8. Если эту цифру поставить на последнее место, то число увеличится на 18. Найдите исходное натуральное число. Ответ: 890
  11. Задано некоторое двузначное число, кратное 3. Если между его цифрами вставить нуль и к полученному трехзначному числу прибавить удвоенную цифру его сотен, то получится число, которое в 9 раз больше, чем исходное двузначное число. Найдите это двузначное число. Ответ: 69
  12. Разность двух чисел равна 48, а разность между средним арифметическим и средним геометрическим этих чисел равна 18. Найти эти числа. Ответ: 49 и 1
  13. Если двузначное число разделить на число, записанное теми же цифрами, но в обратном порядке, то в частном получится 4, а в остатке 15. Если же из данного числа вычесть 9, то получится сумма квадратов цифр, использованных для записи этого числа. Найти это число. Ответ: 91

 

Метки текстовые зада, числа. Смотреть запись.

элементарная теория чисел — Как доказать правило делимости для $ 3 \, $ [выброс троек]

Как насчет индукции?

Это, очевидно, верно для однозначных чисел 3, 6 и 9 долларов, поэтому у нас есть наш базовый случай (на самом деле, только случай 3 долларов — это все, что нужно, но я предпочитаю быть в безопасности, когда он приходит к индукции).

Теперь предположим, что у нас есть число, кратное 3 $, и назовем его $ n $. Мы также можем предположить, что сумма цифр $ n $ делится на $ 3 $.Я хочу показать, что сумма цифр $ n + 3 $ также делится на $ 3 $. Если это так, то мы закончили, потому что принцип индукции позаботится о любом случае за нас оттуда.

Сумма цифр $ n $ — это некоторое число, назовем его $ m $, и предполагается, что это число делится на $ 3 $. Теперь, если нам повезет, сумма цифр в $ n + 3 $ будет всего $ m + 3 $, и под удачей я подразумеваю, что никакого переноса нет. Итак, если добавление $ 3 $ к $ n $ не требует переноса, то все готово.

Если есть перенос, то давайте на секунду представим, что последняя цифра $ n $ может превышать $ 9 $.В этом случае сумма цифр $ n + 3 $ действительно была бы равной $ m + 3 $. К сожалению, это не так, но что на самом деле происходит, когда мы выполняем перенос? Мы вычитаем 10 долларов из цифры 1 доллар и прибавляем 1 доллар к 10 долларам. Это окажет чистое влияние на сумму цифр, которую мы вычтем $ 9 $, так что в этом случае сумма цифр в $ n + 3 $ будет $ m + 3-9 = m-6 $, что по-прежнему делится на $ 3. $, так что нет проблем!

«Держись, не так быстро», — скажете вы. «Что, если добавление 1 доллара к 10-долларовой цифре приведет к переносу?» Что ж, мой просвещенный читатель, в этом случае применим тот же аргумент, что и в предыдущем абзаце, только на один пробел влево в цифрах $ n $.Чистый эффект: сумма цифр $ n + 3 $ составляет $ m-6-9 = m-15 $, но все равно делится на 3 $. Если есть перенос из сотен цифр, мы вычтем еще 9 долларов, что в сумме составит от m до 24 долларов. И так далее. Вы никогда не сделаете такой керри, чтобы взять $ m + 3 $ из пространства, кратного трем. На этом доказательство завершается.

Загадочное число 6174 | plus.maths.org

Март 2006 г.


Каждый может раскрыть тайну

Число 6174 — действительно загадочное число.На первый взгляд это может показаться не таким очевидным. Но, как мы скоро увидим, любой, кто умеет вычитать, может раскрыть тайну, которая делает 6174 таким особенным.

Операция Капрекара

В 1949 году математик Д. Р. Капрекар из Девлали, Индия, разработал процесс, известный теперь как операция Капрекара . Сначала выберите четырехзначное число, в котором все цифры не одинаковы (это не 1111, 2222, …). Затем переставьте цифры, чтобы получить наибольшее и наименьшее числа, которые могут образовать эти цифры.Наконец, вычтите наименьшее число от наибольшего, чтобы получить новое число, и продолжайте повторять операцию для каждого нового числа.

Это простая операция, но Капрекар обнаружил, что она приводит к удивительному результату. Давайте попробуем, начав с числа 2005, цифр прошлого года. Максимальное число, которое мы можем сделать с помощью этих цифр, равно 5200, а минимальное — 0025 или 25 (если одна или несколько цифр равны нулю, вставьте их в левую часть минимального числа). Вычитания:

5200 — 0025 = 5175
7551–1557 = 5994
9954–4599 = 5355
5553 — 3555 = 1998
9981–1899 = 8082
8820 — 0288 = 8532
8532–2358 = 6174
7641–1467 = 6174

Когда мы достигаем 6174, операция повторяется, возвращая каждый раз 6174.Мы называем число 6174 ядром этой операции. Итак, 6174 — это ядро ​​для операции Капрекара, но так ли это особенное, как у 6174? Что ж, 6174 — не только единственное ядро ​​для этой операции, но и еще один сюрприз в рукаве. Давайте попробуем еще раз, начиная с другого числа, например 1789.

9871 — 1789 = 8082
8820 — 0288 = 8532
8532–2358 = 6174

Мы снова достигли 6174!

Очень загадочное число…

Когда мы начали с 2005 года, процесс достиг 6174 за семь шагов, а для 1789 за три шага. Фактически, вы набираете 6174 для всех четырехзначных чисел, у которых не все цифры одинаковы. Это прекрасно, правда? Операция Капрекара настолько проста, но дает такой интересный результат. И это станет еще более интригующим, если мы подумаем о том, почему все четырехзначные числа достигают это загадочное число 6174.

Только 6174?

Из цифр любого четырехзначного числа можно упорядочить максимальное число, расположив цифры в порядке убывания, и минимальное число, расположив их в порядке возрастания.Итак, для четырех цифр a, b, c, d , где

9 ≥ a ≥ b ≥ c ≥ d ≥ 0

и a, b, c, d — не все одинаковые цифры, максимальное число — abcd , а минимальное — dcba .

Мы можем вычислить результат операции Капрекара, используя стандартный метод вычитания, применяемый к каждому столбцу этой задачи:

a б c д
д c б a

А B С D

, что дает отношения

D = 10 + d — a (при a> d)
C = 10 + c — 1 — b = 9 + c — b (поскольку b> c — 1)
B = b — 1 — c (поскольку b> c)
А = а — д

для тех чисел, где a> b> c> d .

Число будет повторяться при операции Капрекара, если получившееся число ABCD можно записать с использованием первых четырех цифр a, b, c и d . Итак, мы можем найти ядра операции Капрекара, рассмотрев все возможные комбинации { a, b, c, d } и проверив, удовлетворяют ли они приведенным выше соотношениям. Каждый из 4! = 24 комбинации дают систему из четырех одновременных уравнений с четырьмя неизвестными, поэтому мы сможем решить эту систему для a, b, c и d .

Оказывается, что только одна из этих комбинаций имеет целочисленные решения, удовлетворяющие 9 ≥ a ≥ b ≥ c ≥ d ≥ 0 . Эта комбинация равна ABCD = bdac , а решение одновременных уравнений: a = 7, b = 6, c = 4 и d = 1. То есть ABCD = 6174. Не существует действительных решений одновременных уравнений, возникающих из некоторых цифр в {a, b, c, d} . будучи равными. Следовательно, число 6174 — единственное число, которое не изменила операция Капрекара — наше загадочное число уникально.

Для трехзначных чисел происходит то же самое. Например, применение операции Капрекара к трехзначному числу 753 дает следующее:

753–357 = 396
963 — 369 = 594
954–459 = 495
954–459 = 495

Число 495 является уникальным ядром для операции с трехзначными числами, и все трехзначные числа достигают 495 с помощью операции. Почему бы тебе самому не проверить?

Как быстро до 6174?

Это было примерно в 1975 году, когда я впервые услышал о числе 6174 от друга, и тогда я был очень впечатлен.Я думал, что будет легко доказать, почему это явление произошло, но я не мог найти причину, почему. Я использовал компьютер, чтобы проверить, все ли четырехзначные числа достигли ядра 6174 за ограниченное количество шагов. Программа, в которой было около 50 операторов на Visual Базовый, проверил все 8991 четырехзначное число от 1000 до 9999, где все цифры не совпадали.

В таблице ниже показаны результаты: каждое четырехзначное число, где все цифры не равны, достигает 6174 в процессе Капрекара, причем не более чем за семь шагов.Если вы не набрали 6174 после семи раз использования операции Капрекара, значит, вы ошиблись в своих расчетах и ​​должны попробовать еще раз!

Итерация Частота
0 1
1 356
2 519
3 2124
4 1124
5 1379
6 1508
7 1980

Какой путь на 6174?

Моя компьютерная программа проверила все 8991 число, но в своей статье Малкольм Лайнс объясняет, что при расследовании операции Капрекара достаточно проверить только 30 из всех возможных четырехзначных чисел.

Как и раньше, предположим, что четырехзначное число — это abcd , где

9 ≥ a ≥ b ≥ c ≥ d ≥ 0 .

Давайте посчитаем первое вычитание в процессе. Максимальное количество — 1000a + 100b + 10c + d , а минимальное — 1000d + 100c + 10b + a . Итак, вычитание:

1000a + 100b + 10c + d — (1000d + 100c + 10b + a)
= 1000 (a-d) + 100 (b-c) + 10 (c-b) + (d-a)
= 999 (a-d) + 90 (b-c)

Возможное значение (a-d), — от 1 до 9, а (b-c), — от 0 до 9.Просматривая все возможности, мы можем увидеть все возможные результаты первого вычитания в процессе. Они показаны в таблице 1.

Таблица 1: Числа после первого вычитания в процессе Капрекара

Нас интересуют только числа, в которых не все цифры равны и

a ≥ b ≥ c ≥ d ,

, поэтому нам нужно рассматривать только те, у которых (a-d) ≥ (b-c) . Таким образом, мы можем игнорировать серую область в таблице 1, которая содержит те числа, где

(а-г) <(б-в) .

Теперь расположим цифры чисел в таблице в порядке убывания, чтобы получить максимальное число, готовое для второго вычитания:

Таблица 2: Максимальные числа, готовые для второго вычитания

Мы можем игнорировать дубликаты в Таблице 2 (серые области), и у нас остается только 30 чисел, чтобы проследить оставшуюся часть процесса. На следующем рисунке показаны маршруты, по которым эти числа достигают 6174.

Как эти 30 чисел достигают 6174

Из этого рисунка вы можете увидеть, как все четырехзначные числа достигают 6174 и достигают его максимум за семь шагов.Несмотря на это, я все еще думаю, что это очень загадочно. Я полагаю, что Капрекар, открывший это число, был чрезвычайно умен или у него было много времени, чтобы обдумать его!

Две цифры, пять цифр, шесть и более …

Мы видели, что четырех- и трехзначные числа достигают уникального ядра, но как насчет других чисел? Оказывается, ответы на эти вопросы не столь впечатляющи. Давайте попробуем это для двузначного числа, скажем, 28:

.

82–28 = 54
54–45 = 9
90 — 09 = 81
81 — 18 = 63
63 — 36 = 27
72 — 27 = 45
54–45 = 9

Не требуется много времени, чтобы убедиться, что все двузначные числа дойдут до цикла 9 → 81 → 63 → 27 → 45 → 9.В отличие от трех- и четырехзначных чисел, для двузначных чисел не существует уникального ядра.

А как насчет пяти цифр? Есть ли ядро ​​для пятизначных чисел, таких как 6174 и 495? Чтобы ответить на этот вопрос, нам нужно будет использовать тот же процесс, что и раньше: проверьте 120 комбинаций {a, b, c, d, e} для ABCDE , так что

9 ≥ a ≥ b ≥ c ≥ d ≥ e ≥ 0

и

abcde — edcba = ABCDE .

К счастью, вычисления уже были выполнены на компьютере, и известно, что не существует ядра для операции Капрекара с пятизначными числами.Но все пятизначные числа попадают в один из следующих трех циклов:

71973 → 83952 → 74943 → 62964 → 71973
75933 → 63954 → 61974 → 82962 → 75933
59994 → 53955 → 59994

Как указывает Малкольм Лайнс в своей статье, проверка того, что происходит с шестью и более цифрами, займет много времени, и эта работа станет чрезвычайно скучной! Чтобы спасти вас от этой участи, в следующей таблице показаны ядра для двух- и десятизначных чисел (подробнее см. Архив Мэтьюза из Развлекательная математика.Похоже, что операция Капрекара сводит каждое число к уникальному ядру только для трех- и четырехзначных чисел.

Цифры Ядро
2 Нет
3 495
4 6174
5 Нет
6 549945, 631764
7 Нет
8 63317664, 97508421
9 554999445, 864197532
10 6333176664, 9753086421, 9975084201

Красиво, но разве это особенное?

Мы видели, что все трехзначные числа достигают 495, а все четырехзначные числа достигают 6174 под действием Капрекара.Но я не объяснил, почему все такие числа достигают уникального ядра. Случайно ли это явление или есть более глубокая математическая причина, почему это происходит? Каким бы красивым и загадочным ни был результат, это могло быть просто случайностью.

Давайте остановимся и рассмотрим красивую головоломку Юкио Ямамото из Японии.

Если вы умножите два пятизначных числа, вы получите ответ 123456789. Можете ли вы угадать два пятизначных числа?

Это очень красивая головоломка, и вы можете подумать, что за ней должна быть спрятана большая математическая теория.Но на самом деле красота это всего лишь случайность, есть и другие очень похожие, но не очень красивые примеры. Например:

(Мы можем дать вам подсказку, чтобы помочь вам решить эти головоломки, и вот ответы.)

Если бы я показал вам загадку Ямамото, вы бы вдохновились на ее решение, потому что она такая красивая, но если бы я показал вам вторую загадку, вы могли бы совсем не заинтересоваться. Я думаю, что проблема Капрекара похожа на головоломку Ямамото отгадывать числа. Нас привлекают оба, потому что они такие красивые.И поскольку они такие красивые, мы чувствуем, что в них должно быть что-то большее, хотя на самом деле их красота может быть просто случайным. Такое недопонимание привело к развитию математики и естествознания в прошлом.

Достаточно ли знать, что все четырехзначные числа достигают 6174 с помощью операции Капрекара, но не знать, почему? До сих пор никто не мог сказать, что все числа, достигающие уникального ядра для трех- и четырехзначных чисел, являются случайным явлением. Это свойство кажется настолько удивительным, что заставляет нас ожидать, что за ним прячется большая теорема теории чисел.Если мы сможем ответить на этот вопрос, мы могли бы найти это просто красивое недоразумение, но мы надеемся, что нет.

Примечание редакции: многие читатели заметили, что многократное сложение цифр любого из ядер операции Капрекара всегда равно 9. Узнайте, почему, в этом продолжении статьи.

Список литературы
  • Капрекар Д. Р., «Другой пасьянс», Scripta Mathematica , том 15, стр. 244–245 (1949)
  • Гарднер, Мартин, «Магические числа доктора Матрицы», японская версия, Токио: Кинокуния (1978)
  • линий, Малкольм Э., Число для ваших мыслей: факты и предположения о числах …, Бристоль: Хильгер (1986)
  • Нишияма, Ютака, Алгоритм Кураши-но, Киото: Наканишия (1993)

    Об авторе

    Ютака Нишияма — профессор Осакского экономического университета, Япония. После изучения математики в Университете Киото он 14 лет проработал в IBM Japan. Он интересуется математикой, которая встречается в повседневной жизни, и написал семь книг по этому предмету.Самая последняя из них, названная «Тайна пяти в природе», среди прочего исследует, почему многие у цветов по пять лепестков. Профессор Нишияма в настоящее время посещает Кембриджский университет.

Безопасность | Стеклянная дверь

Мы получаем подозрительную активность от вас или от кого-то, кто использует вашу интернет-сеть. Подождите, пока мы убедимся, что вы настоящий человек. Ваш контент появится в ближайшее время. Если вы продолжаете видеть это сообщение, напишите нам чтобы сообщить нам, что у вас проблемы.

Nous aider à garder Glassdoor sécurisée

Nous avons reçu des activités suspectes venant de quelqu’un utilisant votre réseau internet. Подвеска Veuillez Patient que nous vérifions que vous êtes une vraie personne. Вотре содержание apparaîtra bientôt. Si vous continuez à voir ce message, veuillez envoyer un электронная почта à pour nous informer du désagrément.

Unterstützen Sie uns beim Schutz von Glassdoor

Wir haben einige verdächtige Aktivitäten von Ihnen oder von jemandem, der in ihrem Интернет-Netzwerk angemeldet ist, festgestellt.Bitte warten Sie, während wir überprüfen, ob Sie ein Mensch und kein Bot sind. Ihr Inhalt wird в Kürze angezeigt. Wenn Sie weiterhin diese Meldung erhalten, informieren Sie uns darüber bitte по электронной почте: .

We hebben verdachte activiteiten waargenomen op Glassdoor van iemand of iemand die uw internet netwerk deelt. Een momentje geduld totdat, мы узнали, что u daadwerkelijk een persoon bent. Uw bijdrage zal spoedig te zien zijn. Als u deze melding blijft zien, электронная почта: om ons te laten weten dat uw проблема zich nog steeds voordoet.

Hemos estado detectando actividad sospechosa tuya o de alguien con quien compare tu red de Internet. Эспера mientras verificamos que eres una persona real. Tu contenido se mostrará en breve. Si Continúas recibiendo este mensaje, envía un correo electrónico a para informarnos de que tienes problemas.

Hemos estado percibiendo actividad sospechosa de ti o de alguien con quien compare tu red de Internet. Эспера mientras verificamos que eres una persona real.Tu contenido se mostrará en breve. Si Continúas recibiendo este mensaje, envía un correo electrónico a para hacernos saber que estás teniendo problemas.

Temos Recebido algumas atividades suspeitas de voiceê ou de alguém que esteja usando a mesma rede. Aguarde enquanto confirmamos que Você é Uma Pessoa de Verdade. Сеу контексто апаресера эм бреве. Caso продолжить Recebendo esta mensagem, envie um email para пункт нет informar sobre o проблема.

Abbiamo notato alcune attività sospette da parte tua o di una persona che condivide la tua rete Internet.Attendi mentre verifichiamo Che sei una persona reale. Il tuo contenuto verrà visualizzato a breve. Secontini visualizzare questo messaggio, invia un’e-mail all’indirizzo per informarci del проблема.

Пожалуйста, включите куки и перезагрузите страницу.

Это автоматический процесс. Ваш браузер в ближайшее время перенаправит вас на запрошенный контент.

Подождите до 5 секунд…

Перенаправление…

Заводское обозначение: CF-102 / 6307db6719667b4f.

Сумма трех последовательных четных чисел равна 138, что является наименьшим из трех чисел

840 = 2 3 × 3 × 5 × 7, очень сложное число, наименьшие числа, делящиеся на числа от 1 до 8… 857 = простое число, сумма трех последовательных простых чисел (281 + 283 …

Список параметров шпинделя Fanuc

  • 4 октября 2019 г. · Ответ: Сумма означает, что вам нужно сложить числа (-5 + -12 = — 5 — 12 = -17). Произведение означает, что вам нужно умножить числа (-5 умножить на -12 = 60). Вопрос: Произведение двух чисел равно 45. Их сумма равна 18. Какие числа? Ответ: два числа — 15 и 3, так как 15 умноженное на 3 дает 45, а 15, добавленное к 3, дает 18.
  • Порядок-3 с наименьшей суммой: эти простые числа не являются ни последовательными, ни в арифметической прогрессии.Два пальмовых магических квадрата: все числа в этой тройке являются 11-значными палиндромными простыми числами. Разность констант порядка 13: вложенные квадраты порядков 13, 11, 9, 7, 5, 3, 1. Двустороннее простое число порядков 7 пандиагональ

Сумма трех последовательных нечетных чисел равна 39 Найдите число Получите нужные ответы , сейчас же! … поэтому 3 обязательных последовательных нечетных числа — это 11, 13 и 15.

Сумма трех чисел. Заявление. Напишите программу, которая берет три числа и выводит их сумму. Каждый номер приводится в отдельной строке.Во всех задачах введите данные с помощью input () и распечатайте результат с помощью print ().

4 октября 2019 г. · Ответ: Сумма означает, что вам нужно сложить числа (-5 + -12 = -5 — 12 = -17). Произведение означает, что вам нужно умножить числа (-5 умножить на -12 = 60). Вопрос: Произведение двух чисел равно 45. Их сумма равна 18. Какие числа? Ответ: Два числа — 15 и 3, так как 15 умноженное на 3 равно 45, а 15, добавленное к 3, равно 18. Сумма трех последовательных четных чисел равна 108. Какое наименьшее из трех чисел? 34.Половину денег Мелани потратила на походы в кино. Она помыла …

Двухэтапные уравнения — задачи со словами — целые числа 1) Сумма трех последовательных четных чисел равна 66. Какое наименьшее из этих чисел? 2) Бет продала половину своих комиксов, а затем купила еще девять. Сейчас у нее 28. Со скольких она начала? 3) 304 студента отправились на экскурсию. Было заполнено семь автобусов, и 17 студентов приехали на автомобилях.

Сумма трех последовательных четных чисел от 84 до 96.а. Напишите неравенство, чтобы найти три числа. Пусть n представляет наименьшее четное число. б. Решите неравенство. математика. сумма трех последовательных чисел равна восемнадцати. Напишите выражение, описывающее уравнение, и решите состояние трех чисел

Датчик скорости carplay

Среднее — это число, которое уравновешивает список чисел As, Есть семь чисел Первые три числа, (Четвертое нет = Среднее), Последние 3 Числа. Четвертое число = 20 наибольшее число (т.е. седьмое число) = 20 + 3 = 23

Это наименьшее число, которое можно выразить как сумму двух разных квадратов двумя разными способами: 65 = 4 2 +7 2 = 8 2 +1 2 .См. Также 50. 65 также является наименьшим числом, квадрат которого выражается как сумма двух разных квадратов более чем двумя способами: 65 2 = 16 2 + 63 2 = 33 2 + 56 2 = 39 2 + 52 2 = 25 2 + 60 2.

Номера на английском языке. Кардинальные числа (один, два, три и т. Д.) — это прилагательные, относящиеся к количеству, а порядковые числа (первое, второе, третье и т. Д.) — к распределению. Чтобы узнать денежную сумму, сначала прочитайте полное число, затем добавьте название валюты.

Какое наименьшее из трех чисел? Мне очень жаль, мне нужна помощь еще раз. Я не понимаю этой проблемы. Это моя последняя проблема. Я получил эту проблему от моего учителя математики в 7-м классе, и я пропустил два дня в школе, и мне нужно набрать 100 баллов. тест и моя домашняя работа, чтобы я мог подбодрить могу…

Например: 2 — ЧЕТНОЕ число, так как мы получаем остаток «0» при делении на «2». Например: 11 — НЕЧЕТНОЕ число, поскольку мы получаем остаток «1» при делении на «2» mymathtables contains разнообразные математические таблицы для печати. ​​Это поможет детям и ученикам узнать основные факты о числах. Таблицы основных чисел красочные и отличный PDF-файл для обучения …

11 сентября 2011 г. · Среднее значение набора последовательных чисел равно посередине между наименьшим и наибольшим числом в наборе.Чтобы получить это, получите сумму наименьшего и наибольшего, а затем разделите ее на. попробуйте ответить на этот вопрос обычным способом, а затем воспользуйтесь сокращенным способом. 1) Найдите среднее значение первых 30 последовательных положительных чисел. 2) Найдите среднее значение первого … Сумма трех последовательных чисел 72. Какое наименьшее из этих чисел? Математика. Предположим: 1-е число [наименьшее число]: X 2-е число: X + 13-е число [наибольшее число]: X + 2 В качестве их суммы … Девять раз число, уменьшенное на 15, равно сумме 3 и В шесть раз больше числа.Найдите номер? Математика Сумма трех последовательных чисел 72. Какое наименьшее из этих чисел? Математика. Предположим: 1-е число [наименьшее число]: X 2-е число: X + 13-е число [наибольшее число]: X + 2 В качестве их суммы … Девять раз число, уменьшенное на 15, равно сумме 3 и В шесть раз больше числа. Найдите номер? Математика

16 декабря 2009 г. · Пусть x будет наименьшим целым числом. Поскольку они являются последовательными четными, разница между ними равна 2, поэтому второе целое число равно x + 2, а третье — x + 4.Сумма их квадратов равна 440.

Я призываю вас продолжать попытки, выберите одно четное число, затем другое четное число, сложите их вместе, и вы увидите, что вы продолжаете получать четные числа. И это имеет смысл. Потому что, если у вас есть одно число, кратное двум, и вы добавляете его к другому числу, кратному двум, имеет смысл, что сумма будет кратной двум.

Xbox game pass ultimate mod apk

Таблица dpercent27architecte moderne

  • АНАЛИЗ.В этой программе на C для поиска суммы чисел с использованием примера цикла while, цикл while используется для итерации между 1 и введенным пользователем значением. Если вы не знаете цикл While Loop, обратитесь к статье While Loop in C для получения дополнительной информации.

    Карла учится в США. Она покупает подержанные учебники, потому что у нее ограниченный бюджет (совет от профессионала: никогда не покупайте новые книги)! Она купила книгу по математике и книгу для чтения на общую сумму 54 доллара. Цена учебника по математике на 8 долларов превышает стоимость книги для чтения более чем в 3 раза.Сколько стоит каждая книга?

  • Порядковый номер получается добавлением 3 к предыдущему целому числу. Пример 2. Найдите недостающие термины в следующей последовательности. Чтобы найти образец этой последовательности, исследуются три последовательных числа 24, 28 и 32, и получается правило.

    5 августа 2020 г. · Учитывая массив из N чисел, нам нужно максимизировать сумму выбранных чисел. На каждом шаге вам нужно выбрать число A i, удалить одно его вхождение и удалить все вхождения A i-1 и A i +1 (если они есть) в массиве.Повторяйте эти шаги, пока массив не станет пустым. Проблема в том, чтобы максимизировать сумму выбранных чисел.

Последние вакансии в области управления строительством на октябрь 2020 года Эфиопия

  • Номера на английском языке. Кардинальные числа (один, два, три и т. Д.) — это прилагательные, относящиеся к количеству, а порядковые числа (первое, второе, третье и т. Д.) — к распределению. Чтобы узнать денежную сумму, сначала прочитайте полное число, затем добавьте название валюты.

    Числа, следующие друг за другом в порядке, без пробелов, от наименьшего к наибольшему.12, 13, 14 и 15 — последовательные числа. 22, 24, 26, 28 и 30 — последовательные четные числа. 40, 45, 50 и 55 являются последовательными числами, кратными 5.

Как начать бизнес в Техасе SP101 357 4

Определение водной экосистемы Oracle a dios por la salud de mi madre

Surah yaseen urdu tarjuma

Безопасность Windows подключение смарт-карты Складывающееся крыло самолета

Двойное пространство apk
Текстуры боевой зоны не загружаются ПК

Стоимость восстановления S52

Forge of empires chateau frontenac стоит

Frigida gallery свет горячей поверхности остается на

Какое наименьшее из этих чисел? Произведение y и квадрата суммы x и 5 1.Колония муравьев вторгается в карамель в кондитерской. В первый день они съедают 14 порций карамели, во второй день — 12 порций карамели, в третий день — 34. a. Wha.Sep 11, 2011 · Среднее значение набора последовательных чисел находится посередине между наименьшим и наибольшим числом в наборе. Чтобы получить это, получите сумму наименьшего и наибольшего, а затем разделите ее на. попробуйте ответить на этот вопрос обычным способом, а затем воспользуйтесь сокращенным способом. 1) Найдите среднее значение первых 30 последовательных положительных чисел. 2) Найдите среднее значение первого…
Манипур сатта ма
Apple TV следующего поколения

Упрощающий квадратный корень с множественным выбором

Модификации оружия Ravenfield beta 5

Книги самопомощи для чтения в 2020 году

1). Суммарный возраст 4 членов семьи 5 лет назад составлял 94 года. Сегодня, когда дочь выдана замуж и заменена невесткой, их суммарный возраст составляет 92 года. Если допустить, что никаких других изменений в структуре семьи не произошло и все люди живы, сколько лет дочери и невестки? 31 декабря 2020 г. — сумма трех последовательных нечетных чисел и трех последовательных четных чисел вместе составляет 231.Кроме того, наименьшее нечетное число на 11 меньше наименьшего четного числа. Какова сумма наибольшего нечетного числа и наибольшего четного числа? A) 82b) 83c) 74d) Нет Правильный ответ — вариант ‘D’.
Watch rambo_ first blood part 1
Разблокированные игры wtf granny

Marlex fittings

Melody Personal Music Assistant apk

Toot iranian Fruit

23 ноября, 2016 · сумма 4 последовательные (целые) числа: x + x + 1 + x + 2 + x + 3.В сумме получается 4x + 6. Мы можем вынести 2: 4x + 6 = 2 (2x + 3), что явно делится на 2 и, следовательно, даже. 3/8. три восьмых. 21/1000. двадцать одна тысячная. Дроби используются во всех языковых стилях, включая научный и технический английский. Примеры: Полпинты пива Четверть секунды Три четверти мили Три пятых содержимого бутылки. Толщина одна тридцать секунд дюйма.
Squashfs mount
Icumsa 45 Sugar wikipedia

Включить битлокер Windows 10 в командной строке

Scaqmd rules

Cmc course usmc

Нечетные числа в порядке убывания и четности порядок.Найдите количество вхождений числа в заданном отсортированном массиве. Найдите все общие числа в данных трех отсортированных массивах. Найдите первый повторяющийся элемент в массиве по его индексу; Минимальное количество, которое не может быть сформировано какой-либо подмножеством массива
440 гидравлических роликовых кулачков
Полицейский участок Деонар

Таблица цветов раствора Mapei

Обучение инспекторов строительства Icc

Модификации Outwitt

Следовательно, три последовательных числа, сумма которых равна 156, на самом деле являются 50, 52 и 54, а 50, очевидно, является наименьшим.Связанные вопросы. Ответ: — наименьшее из трех четных чисел, в результате чего получается 156, равно 50.

Bowflex max trainer m5 parts canada Как испортить кому-то отказано в подключении

Автокатастрофа округа Буллох

D
Устранение неисправностей Smart Board 6065

Причины перегрева компрессора холодильника

Ansible grub_cmdline_linux

Примеры: целые числа; натуральные числа; простые числа; четные числа; нечетные числа; большие числа; круглые числа; отрицательные числа; число 12; число двенадцать; номер 165.Около 60 процентов суммы предоставил спонсор. Leech cap wow
Uno platform github

Fwhm matlab

Ano ang wikang panturo ng pilipinas
3

Слайд-шоу колледжа выборщиков

Программа для проверки того, является ли номер EVEN. В этой программе на C мы прочитаем целое число, и программа проверит, является ли данное целое число ЧЕТНЫМ или НЕЧЕТНЫМ. Программа для поиска наибольшего числа среди трех чисел.В этой программе на C мы прочитаем от пользователя три целых числа и выведем из них наибольшее число. Наименьшее число — 20, а наибольшее — 27. (27-20) + 1 = 8. Восемь чисел составляют 4 пары, а сумма каждой пары равна 47. 4 x 47 = 188. Ue4 получить первая буква строки
Краткая история канадского Дня благодарения

Mossberg 930 waterfowl black

Три листа ралли
Щенки английского мастифа Вирджиния

Pymssql зашифрованное соединение

c Beretz34 vs Beretz34 Цепи пожарной сигнализации

Планы навеса 5×8

Kibana sso oauth
6

Конструкция схемы Finfet

Ascendant shard destiny 2
Пузырьковая сортировка во время цикла

2 Waco Texas Дресс-код для подготовки к колледжу Джонса
Видео Cny о фертильности

Калькулятор стероидов

Какое наименьшее из трех чисел? Щелкните здесь, чтобы увидеть ВСЕ проблемы в программе Numbers Word.Наименьшее число — 44. Пусть три последовательных четных числа будут 2n-2, 2n и 2n + 2. Итак, это числа 44, 46, 48. Когда у вас есть последовательные числа и некоторая информация об их сумме, взяв среднее значение, вы попадете прямо в середину из них.

Платите кому-нибудь с помощью PayPal Альбомы штампов Харриса

Ford f150 Ширина оси 8,8
Панель Zinsco
Наиболее распространенные числа trifecta в скачках
Работает ли клей gorilla82

Ertugrul ghazi, сезон 2, серия 77 на урду, дублированный

Купон на грузовой трейлер в гавани 1090

Mujoco xml geom

Последовательные числа — это числа, которые следуют друг за другом в порядке от меньшего к большему, при регулярном подсчете. представляют последовательные числа алгебраически, пусть одно из чисел будет x.Затем следующий подряд Какие числа? Три последовательных числа, кратных пяти, составляют 75.

Шаблон политики резервного копирования и восстановления данных Примеры плана профессионального роста для библиотекарей

Какое из следующих утверждений описывает эукариотические клетки? нагрузка на открытом воздухе
Комбинированный пакет вспомогательного усилителя
Бесплатный вспомогательный бот

Программа автоматических закупок American Express

Stiiizy benzene
2

Инженер-программист с капиталом одна зарплата

Redmi 7 flash без авторизованного аккаунта

Убийство в Монро-Ла

Si документ 2595024
Asus ddns down

Roll a ball game 3d unity 5

Unscramble alluded

Spartanburg sc Дома в аренду

West Монро, Лос-Анджелес Поли ce Департамент 2301 N 7th St West Monroe, LA 71291.Обратитесь в службу экстренной помощи: 9-1-1. В неэкстренных случаях: 318-396-2722. Отдел уголовных расследований Пола Блански [адрес электронной почты защищен] 318-397-6747 KNOE 8 News, Монро, Луизиана. 170 061 лайк · 28 890 говорят об этом. Это официальная страница новостей KNOE 8 на Facebook!
Векторы физика ppt
Кнопка ввода на пульте

Лучшие охоты на техасских оленей

Документы Ugsn

Тактический Hk45 против тактического USP

Спарта преступность приводит к стрельбе в Ла Кросс.Чт, 15.08.2019 … Вестник округа Монро. Evans Print & Media Group. 1302 River Rd. P.O. Вставка 252 «Ипс, убившие Мэрилин Монро», «Майкл Джексон», «Элвис Пресли и принцессу Диану». pontiac turbo kits, жгут проводов ford windstar 2003 года, комбинированная электрическая схема розетки переключателя, электрическая схема для пикапа 1981 gmc, схема предохранителей 95 saturn, электрическая схема s10 wwwjustanswercom chevy 3txkjchevys10, lucid diagrama de cableado de la de la …
Модульные дома на продажу в Небраске
Lg tv setup menu

Tracking sheet excel

Connexx smart relay

Теплопроводность через твердый цилиндр

Просмотрите объявления Oodle Monroe, LA, чтобы найти все, что вам нужно .Поиск объявлений на Oodle очень прост благодаря широким параметрам и критериям поиска. Если вы хотите продать что-то в Монро, штат Лос-Анджелес, разместите это на Oodle. Мэрилин Монро была известной и одним из самых выдающихся и самых больших секс-символов Голливуда. Эта биография Мэрилин Монро предоставляет подробную информацию о ее детстве, жизни, достижениях, работе и хронологии.
Материнская плата HP prodesk 400 g5 sff
Грибная ферма на продажу pa

Likee login pc

98 civic idle boost

Ограничитель оборотов спортсмена Polaris

Полиция Монро расследует смерть двое мужчин, которые были смертельно ранены во вторник в результате стрельбы в апартаментах Parkview на Richwood Road No.2. Полиция отреагировала на сообщения о стрельбе во вторник около 7:30 утра. Прибыв в апартаменты, полиция обнаружила на стоянке двух мертвых мужчин. У обеих жертв были видимые огнестрельные ранения. Гондурас Защитник окружающей среды из числа коренного населения убит во время последнего нападения. «Убийство журналиста не убьет историю» Репортеры запускают проект картелей в Мексике.
Sikinde ngoma ya ukae mp3 download
Принятие подарков от фей

Адаптер с 6-контактным разъемом

Предварительный просмотр нотной книги

Шум скрежета элитной посудомоечной машины Kenmore

Courtyard Monroe Airport, Monroe Picture : Убитый таракан — Посмотрите 2269 откровенных фото и видео участников Tripadvisor об аэропорте Кортъярд Монро. В память о Реджинальде Хилле-младшем.«Р.Джей» Реджинальд «Р. Джей» Хилл-младший, 19 лет из Монро, Лос-Анджелес. умер 9 октября 2019 года. Службы будут в 11:00 утра, в субботу, 19 октября, в Большой Новой Антиохийской баптистской церкви, Монро, штат Луизиана.

Что такое сумма цифр

Профессиональный FM-передатчик
Короткая молитва о защите семьи и руководстве

Снимаю шляпу со смайликами facebook

Z490 hackintosh

Eb1a premium processing

A Алгоритм контрольной цифры вычисляет контрольную цифру на основе исходной строки символов, такой как номер счета.Приемник пересчитывает контрольную цифру, чтобы проверить точность ввода данных. Если пересчитанная строка символов содержит правильную контрольную цифру, данные не содержат ошибок и могут быть использованы. 10 ноября 2018 г. · Цифровые суммы. Цифровая сумма числа находится путем сложения всех цифр числа. Если сумма больше одной цифры, мы снова складываем все цифры, пока не получим однозначный ответ.
Настройки анимации ios 13
Как изменить размер в procreate без потери качества

Игрушечный пистолет, стреляющий воздухом

Комплект клапана канистры Колера lowepercent27s

Комплект модернизации Ipod classic

цифровая сумма — это конечный результат многократного вычисления суммы цифр до получения однозначного ответа.Цифровая сумма числа n обозначается как DigitSum (n). Вот иллюстрации свойств арифметики суммы цифр. Первые довольно обыденные, но вторые по праву можно охарактеризовать как потрясающие. Таким образом, сумма всех трехзначных чисел, делящихся на 6, равна 82350. Примечание. Метод, описанный выше, применим не только для нахождения суммы всех трехзначных чисел, делящихся на 6. Этот же метод можно применить для нахождения суммы всех трех чисел. цифры числа, делящиеся на любое число, скажем «k».
Самодельные прицепы для кемперов
Ремонт настенных часов Seth Thomas

2017 amc 10a

Забытый код доступа к экрану Apple

Цена сенсорного экрана Samsung s6 edge

Сумма цифр числа Аннотация .50-74 и 8 = 2 + 6. Если даны (100-74), какова сумма цифр? *
Corgi rescue texas
Шаблон списка аминокислот

Примеры трубопроводов Jenkins

По сравнению с красным голубым светом имеет более высокую частоту и

Ячейки и их рабочий лист среды отвечает

24 сентября 2009 г. · 100 001 сумма = 2. 100 002 сумма = 3. Пока мы меняем только одну цифру, сумма цифр будет увеличиваться на 1 каждую когда число увеличивается на 1.Итак, мы чередуем шансы и эвенты, и первые десять чисел (от 100 000 до 100 009) дадут нам суммы от 1 до 10: 5 шансов, 5 эвенов. Когда цифра десятков увеличивается на 1, давайте посмотрим, что произойдет:
Actiontec g1100
Статуя орла Minecraft

Ghk m4 drum mag

Madden 20 исчезли бесплатные агенты

Срок действия лицензии Cisco Smart истек

Исходное число соизмеримо 6 и 7, используются все числа от 1 до 9, и после четырехкратного округления сумма неотокругленных чисел равна 24.10-значное число Найдите 10-значное число, где первая цифра определяет количество нулей в этом числе, вторая цифра — количество цифр 1 в этом числе и т. Д. Сумма двух цифр = 9. При замене цифр местами , получившееся новое число больше исходного числа на 27. Предположим, что цифра единиц place = x. Тогда разряд десятков будет = (9 — x) Таким образом, двузначное число будет 10 (9 — x) + x. Давайте перевернем цифру. число становится 10x + (9 — x) согласно данному…

Решили? Вы умнее десятилетнего сингапурца? | Математика

Сегодня я задал вам десять вопросов с Международного конкурса математиков в Сингапуре в этом году. Вот вопросы и ответы. В целом у вас все получилось — умнее 10-летнего сингапурца! (С оговоркой, что у них не было ответов с несколькими вариантами ответов, а их всего десять). Единственные вопросы, на которые ваш самый популярный ответ неправильный, — это 6 и 8.(C в Q6 и B в Q8). Спасибо за участие — теперь просмотрите свои наработки …

Для учеников 5 класса:

1. Мэри отрезала 2/5 веревки. Позже она отрезала еще 14 м. Отношение длины оставшейся струны к общей отрезанной длине составляет 1: 3. Какова длина оставшейся струны?

  • A. 5 м
  • Б. 7 м
  • C. 10 м
  • Д. 14 м

Решение — C. [73 процента читателей правильно поняли]

О Мэри! Вот как я бы решил это, используя уравнения. Пусть L будет исходной длиной строки, а R будет тем, что останется после того, как вы дважды разрежете строку. Мы знаем, что R = (L x 3/5) — 14m, и что ((L x 2/5) +14) / R = 3, или 2L / 5 + 14 = 3R. Подставляя первое уравнение во второе, получаем 2L / 5 + 14 = 9L / 5 –42. Что переставляется на: 7L / 5 = 56 или L = 40. Итак, R = 10м.

Интересно, что сингапурский метод решения другой.Это требует от нас более визуального восприятия строки: мы разрезаем ее на 2/5. Затем 14 м, и остается кусок, размер которого составляет треть от того, что было вырезано. Другими словами, у нас остается 1/4 исходной длины. Чтобы сравнить дроби 2/5, а затем 1/4, давайте изменим их на наименьший общий знаменатель, равный 20. Итак, мы отсекаем 8/20, вычитаем 14m и получаем 5/20. Давайте теперь нарисуем строку, разделенную на двадцатые части:

14 метров должны составлять 7/20 длины, что означает, что каждая двадцатая часть равна 2 метрам.Оставшийся кусок веревки составляет 5/20, т.е. 10 м

2. Площади граней прямоугольной коробки составляют 84 см 2 , 70 см 2 и 30 см 2 . Какой объем коробки?

Фотография: ISMC
  • A. 300 см 3
  • Б. 420 см 3
  • C. 490 см 3
  • Д. 504 см 3

Решение: B . [85 процентов читателей правильно поняли]

Сначала нам нужно определить возможные длины сторон, посмотрев, какие два числа умножаются на площадь каждой грани.Грань 84 может быть 1 x 84, 2 x 42, 3 x 28, 4 x 21, 6 x 14 или 7 x 12. Грань 70 может быть 1 x 70, 2 x 35, 5 x 14 или 7 x 10. Грань 30 может быть 1 x 30, 2 x 15, 3 x 10 или 5 x 6.

  • Общие множители между 84 и 70 равны 1, 2, 7 и 14.
  • Общие множители между 84 и 30 равны 1, 2, 3 и 6.

Единственный способ получить 84 с каждым из этих общих множителей — это 14 от верхней строки и 6 снизу. Таким образом, край, граничащий с гранями 84 и 70, имеет длину 14, а край, ограничивающий края 84 и 30, имеет длину 6.Это означает, что высота должна быть 30/6, или 70/14 = 5. Таким образом, объем 14 x 6 x 5 = 420 см.

3. Всего четыре числа. Если пропустить какое-либо одно число, среднее значение оставшихся трех чисел будет 45, 60, 65 или 70. Какое среднее значение всех четырех чисел?

Решение — C . [82 процента читателей правильно поняли]

Если четыре числа — A, B, C и D, то мы знаем, что

  • A + B + C = 45 × 3
  • A + B + D = 60 × 3
  • A + C + D = 65 × 3
  • B + C + D = 70 × 3

Теперь сложите их, чтобы получить 3 A + 3 B + 3 C + 3 D = (45 + 60 + 65 + 70) × 3

Это A + B + C + D = (45 + 60 + 65 + 70) = 240 . Таким образом, их среднее значение составляет 240/4 = 60

4. Марш проходит по улицам от школы (S) до общественного центра (CC). Одна из улиц закрыта. Если марш может идти только на восток или юг, каково количество различных способов добраться до общественного центра? [10 процентов]

Фотография: ISMC

Решение — B. [46 процентов читателей правильно поняли]

Есть 19 возможных способов:

5. Салли дали набор из 5 карточек, пронумерованных от 1 до 5. Петру также дали набор из 5 карточек с номерами от 1 до 5.Затем им завязали глаза и попросили выбрать карту из своих наборов. Сумма чисел на двух карточках была сообщена только Салли, а произведение чисел — только Питеру. Затем им было предложено угадать два числа. Вот что сказал каждый из них:

Питер: Я не знаю двух чисел.

Салли: Теперь я знаю два числа.

Питер: Я до сих пор не знаю двух чисел.

Салли: Позвольте помочь вам.Число, которое мне сказали, больше, чем число, которое вам сказали.

Питер: Теперь я знаю два числа.

Какие два числа?

  • А. 1 и 4
  • Б. 1 и 5
  • C. 2 и 4
  • Д. 2 и 5

Решение — A. [72 процента читателей правильно поняли]

Поскольку каждому из них были даны числа от 1 до 5, если бы Питеру сказали любое из следующих чисел, он мог бы сказать, что это за Были выбраны два числа:

  • 1 → 1 × 1 Два числа — 1 и 1.
  • 2 → 1 × 2 Два числа — 1 и 2.
  • 3 → 1 × 3 Два числа — 1 и 3.
  • 5 → 1 × 5 Два числа — 1 и 5.
  • 6 → 2 × 3 Два числа — 2 и 3.
  • 7 → 1 × 7 Два числа — 1 и 7.
  • 8 ​​→ 2 × 4 Два числа — 2 и 4.
  • 9 → 3 × 3 Два числа — 3 и 3.
  • 10 → 2 × 5 Два числа — 2 и 5.
  • 12 → 3 × 4 Два числа — 3 и 4.
  • 15 → 3 × 5 Два числа — 3 и 5.
  • 16 → 4 × 4 Два числа — 4 и 4.
  • 20 → 4 × 5 Два числа — 4 и 5.
  • 25 → 5 × 5 Две цифры — 5 и 5.

(Продукты 11, 13, 14, 17, 18, 19, 21, 22, 23 и 24 не могут быть сформированы.)

Единственное неоднозначное произведение — 4, поскольку 4 может быть равно 1 × 4 или 2 × 2. Таким образом, когда Питер сказал, что он не знает чисел, Салли сможет узнать, что произведение, которое, по словам Питера, должно быть 4.Поскольку Салли сказала, что сумма, которую ей сказали, больше, чем произведение, которое было сказано Питеру, два числа, которые были выбраны, должны были быть 1 и 4 (сумма = 5), а не 2 и 2 (сумма = 4).

Для учеников 6 класса:

6. Есть 4 ключа и 4 замка. Какое максимальное количество раз вам нужно попробовать замки, чтобы все 4 ключа совпали с их замками?

Решение — B. [30% читателей правильно поняли]

Если вы попробуете 3 ключа на замке 1 st , вы узнаете, что 4 -й ключ совпадает.Итак, вам нужно максимум 3 попытки. Точно так же для блокировки 2 и требуется максимум 2 попытки, а для блокировки 3 rd требуется только 1 попытка. Таким образом, необходимо 3 + 2 + 1 = 6 попыток.

7. На диаграмме (не в масштабе) наклонная линия делит площадь прямоугольника в соотношении 1: 6. Каково соотношение a : b ?

Фотография: ISMC
  • A. 2: 3
  • Б. 1: 2
  • С. 2: 5
  • Д. 1: 3

Решение — C. [63 процента читателей правильно поняли]

Верхний прямоугольник имеет площадь 2 единицы 2 , нижний прямоугольник имеет площадь 6 — 1 = 5 единиц 2 . Мы знаем, что отношение площади верхнего прямоугольника к площади нижнего прямоугольника составляет 2: 5. Поскольку оба прямоугольника имеют одинаковую ширину, тогда a и b также должны быть в соотношении 2: 5.

8. Сколько раз часовая и минутная стрелки часов образуют прямой угол друг с другом между 06:00 и 12:00 в один и тот же день?

Решение: A. [39 процентов читателей правильно поняли]

Время, когда часовая и минутная стрелки часов образуют прямой угол друг с другом в период с 06:00 до 12:00, составляет примерно 0617, 0649, 0722, 0754, 0828, 0900. , 0933, 1005, 1038, 1111, 1149.

Обратите внимание, что это происходит дважды в час, за исключением периода между 0800 и 1000, когда это происходит только три раза, а не четыре раза, как ожидалось. Это потому, что в 09:00 стрелки образуют прямой угол. Таким образом, с 06:00 до 12:00 это происходит (6 × 2) — 1 = 11 раз.

9. В школе 6 учеников 240 человек. Отношение количества студентов, которым нравится наука, к количеству студентов, которые не любят математику, составляет 5: 3. Отношение количества студентов, которым нравится математика, к количеству студентов, которые не любят, составляет 7: 5. Количество студентов, которым нравится и наука. и математике — 86. Сколько студентов не любят и естественные науки, и математику?

Решение n — это B . [75 процентов читателей правильно поняли]

Лучший способ решить эту проблему — использовать диаграмму Венна:

Теперь мы можем записать некоторые суммы:

Если количество студентов, которые любят науку, разделить на количество студентов, которые этого не делают, — 5/3, затем 5/8 от общего числа любят естественные науки, а 3/8 — нет.Поскольку общее количество студентов составляет 240, то количество тех, кто любит науку, равно (5/8) x 240 = 150

Итак, количество студентов, которым нравится только , естествознания = 150-86 = 64

Аналогично числу студентов, которым нравится математика = (7/12) x 240 = 140

Итак, количество студентов, которым нравится только , математика = 140 — 86 = 54

Таким образом, количество студентов, которым не нравятся и естественные науки, и математика, составляет 240 — (64 + 86 + 54) = 36

10.